Содержание
«Почему нельзя сделать вечный двигатель из электродвигателя и генератора, который он вращал бы и питал сам себя?» — Яндекс Кью
Популярное
Сообщества
Стать экспертом Кью
ФизикаНаукаКак это работает?
Роман Иванов
·
26,7 K
Ответить6Уточнить
Стройка века
843
Журнал об экономике, технических и естественных науках в ретроспективе. · 8 авг 2020 · century-arch.ru
Отвечает
Николай Вододохов
Потому что всегда существуют потери на трение и сопротивление катушек. Плюс, закон сохранения энергии, тут как ни крути избежать этого не получится. Тем более какой смысл в установке неспособной совершать полезную работу?
Ответ о интеллектуального журнала Стройка Века. Подписывайтесь на нашу рассылку
Перейти на t.me/century_arch
1 эксперт согласен
Комментировать ответ…Комментировать…
Сергей Касьянов
Физика
38
Электричество, НКУ, инструменты.
· 23 сент 2021
Доброго времени суток, Роман) Предположу: КПД обоих устройств хоть и высок, но не 100%. Даже если собрать такую установку и пренебречь расходами на её запуск, потери будут такие, что эта установка перестанет выдерживать даже собственный темп — двигатель не сможет крутиться столь быстро, чтобы геннратор выдавал нужное напряжение. Но даже если гипотетически эта установка… Читать далее
Комментировать ответ…Комментировать…
Валерий Качан
22
Радиоэлектроник, компьютерщик, Linuxоид, Arduinщик, любитель Raspberry PI, автоматизации… · 8 окт 2018
Самое главное правило физики: ничто из ниоткуда не берется, и ничто вникуда не пропадает — закон сохранения энергии. Вы когда-нибудь задумывались о том, почему греется телефон, или почему греется лампочка? Это ток. КПД. Когда ток идет по проводам, он рассеивается в тепло, тепло-тоже энергия. Сварка — малое количество вольт, но 200-500 ампер. Буквально все амперы уходят… Читать далее
Комментировать ответ…Комментировать…
Первый
Surfer
4
Изучение Вселенной · 3 дек 2020
Ответ прост: Затраты энергии на работу электродвигателя для вращения генератора будут больше, чем выработается генератором и поступит на электродвигатель… Все остальные комментарии — не по теме…
Комментировать ответ…Комментировать…
Первый
Юрий Баскаков
1
30 авг 2021
Да почему нельзя. Все в один голос говорят нельзя. Раньше на самолётах не летали и много лет все говорили что человек летать не сможет. В итоге и летаем и под водой плаваем и телефоны придумали. И все это реально.
Принцип этой установки должен быть просто просчитан. Палиспасы, редуктора уменьшающие силу вращения. То есть собираем схему. Генератор-коробка отбора мощности… Читать далее
1 эксперт не согласен
Виктор Зосимов
возражает
30 августа 2021
Пусть соберёт и покажет, тогда отзову возражение
Комментировать ответ…Комментировать…
виктор к.
5
16 сент 2020
Кто изготовит первым бааллоныиз угле пластикаграфеновые емкости и поместавтомобильможет брать энергию из воздуха более10 лет Новосёлов Петрик рассматривают эти вопросы но ни один из них не хочет внедрять в вРоссии гнать под жопу мешалкой в глазах кто денег даст а внедрять прийдут другие
Комментировать ответ…Комментировать…
Первый
виктор к.
5
16 сент 2020
Известно что всем управляет государство какие проблемы сделать БТГпосмотрите в Интернете этих способов множество но государство бдит наблюдает кто первый познаёт способыполучения свободной энергии тото и получит первый пинок Путин давно говорит да мало что делает последнее сообщение новые технологии попытаются открыть после 2030 года а иначе получите штраф ориентиров… Читать далее
Комментировать ответ…Комментировать…
Алексей Олегович М.
6
9 мая 2020
Нельзя только потому, что любой генератор может выработать столько энергии, сколько получил извне в первичном виде. То есть, если КПД генератора = 100%, то электроэнергии он может выдать не больше, чем получил механической.
Любой двигатель, имея 100% КПД, может выдать механической энергии не более, чем получил электрической.
То есть, построить замкнутую по двум (электрич. .. Читать далее
Комментировать ответ…Комментировать…
Первый
Юрий Гононов
4
11 окт 2019
Я так понимаю что автор задал немного неправильно вопрос, если електродвигатель, будет врашать генератор с одного конца и валом передавать энергию на что то еше например на турбину, но в цепи так же участвует накопитель энергии например аккумулятор, который в свою очередь постоянно питается от генератора, компенсируя затраты энергии аккумулятрра на расскручивание… Читать далее
Arcady Kuligin
21 июня 2020
Бесплатное электричество получают за счет силы гравитации
Комментировать ответ…Комментировать…
Первый
Матвей Фомичёв
9 окт 2018
1. Когда уважаемый, но пожилой учёный утверждает, что что-то возможно, то он почти наверняка прав. Когда он утверждает, что что-то невозможно, — он, весьма вероятно, ошибается.
2. Единственный способ обнаружения пределов возможного состоит в том, чтобы отважиться сделать шаг в невозможное.
3. Любая достаточно развитая технология неотличима от магии.
(Законы Артура… Читать далее
Евгений Перегуда
10 октября 2018
> Известен случай, когда Архимед с помощью своих полиспастов (также именуемых блоками) ОДНОЙ РУКОЙ (!!!) сдвинул с… Читать дальше
Комментировать ответ…Комментировать…
Вечный двигатель | Физика
Среди детских игрушек можно встретить забавный прибор под названием «птичка Хоттабыча» (рис. 68). Птичка периодически наклоняется, опускает клюв в воду и поднимает его. Кое-кто, увидев эту игрушку впервые, может воскликнуть: «Вечный двигатель!» Но… не будем торопиться. Разберем секрет птички. Ее голова и клюв покрыты ватой. Если эту вату слегка увлажнить, то при испарении воды голова птички будет охлаждаться. В нижней ампуле прибора налит эфир или другая легко испаряющаяся жидкость, пары которой заполняют все пространство. При охлаждении головы давление этих паров уменьшается. Тогда под избыточным давлением паров эфира в нижней ампуле эфир поднимается по трубочке, голова птички оказывается тяжелее и наклоняется к стаканчику. Как только жидкость достигнет конца трубочки, пары эфира из нижней ампулы прорвутся в верхнюю (головку), давление паров сравняется, жидкость потечет вниз, птичка вновь поднимет клюв. Не будь испарения влаги, не происходили бы все описанные явления. При испарении затрачивается энергия, которая берется от воды комнатной температуры и из окружающего воздуха. А вечный двигатель должен работать без затраты энергии. Поэтому «птичка Хоттабыча» не является вечным двигателем.
Все заводные игрушки и механизмы работают до тех пор, пока имеют запас потенциальной энергии закрученной пружины. Потенциальная энергия закрученной пружины в процессе работы превращается в другие виды энергии и передается другим частям механизма.
Великий закон природы — закон сохранения и превращения энергии — был открыт в середине XIX века. Но не сразу этот закон получил всеобщее признание. Уж очень заманчива была мысль построить такую машину, которая работала бы сама собой. Бесчисленные проекты всевозможных «вечных двигателей» предлагались людьми, недостаточно знакомыми с основами физики. Даже в наше время встречаются такие «изобретатели».
Представьте себя в роли конструктора бюро изобретений и выясните, в чем заключается ошибка в приводимых ниже проектах вечных двигателей.
При разборе проектов не забывайте следующее. «Вечный двигатель» должен работать без подвода энергии. Создание «вечного двигателя» было бы чудом, но чудес не бывает. Если механизм, выдаваемый за «вечный двигатель», совершает работу, которая состоит в преодолении трения только в частях этого механизма, значит, он откуда-то получает энергию. Надо выяснить откуда. Все проекты «вечных двигателей» можно разбить на две группы: а) проекты, авторы которых не обнаруживают внешних источников энергии (см. «птичку Хоттабыча») и б) проекты, содержащие ошибки, так как авторы проектов недостаточно хорошо знают законы физики.
Итак, познакомимся с некоторыми проектами «вечных двигателей». Будьте внимательны!
1. Через блок перекинута цепь (рис. 69, а, б). Свешивающаяся правая часть цепи под действием силы тяжести переходит с одного уровня на другой. Изобретатель уверен, что спущенная часть может быть снова поднята вверх, если связать концы цепи. Будет ли правая часть цепи перетягивать левую?
2. Два одинаковых шара уравновешены в воздухе на коромысле весов. Если один шар опустить в сосуд с водой, то вода будет выталкивать его и коромысло наклонится (рис. 69, в). Изобретатель утверждает, что шар, выйдя на поверхность воды, снова приобретет свой вес, снова будет опускаться в воду, а коромысло вечно будет качаться. Проверьте проект на опыте и найдите ошибку в проекте. Подумайте, не совершает ли этот «двигатель» работу по преодолению силы трения и вязкости жидкости.
3. Вытекающая из водопроводного крана вода ( рис. 69, г) должна создать в герметически закрытом водонапорном баке пустоту. Автор убежден, что атмосферное давление будет вечно пополнять убыль воды в баке.
4. Самозаводящиеся часы. При повышении температуры окружающего воздуха жидкость в радиаторе и соединенном с ним цилиндре будет расширяться и поршень, находящийся в цилиндре, придя в движение, поднимет гирю часов. Подъем гири не мешает ходу часов. Часовой механизм приводится в движение опускающейся гирей (рис. 69, д).
5. Магнитный вечный двигатель. Железный шарик, притягиваясь магнитом, поднимается по наклонной плоскости. Если в верхней части наклонной плоскости сделать отверстие (рис. 69, е), то шарик, провалившись через отверстие, упадет на желоб, скатится по нему и, обладая запасом кинетической энергии, снова попадет на наклонную плоскость, и так без конца.
6. Обычный предмет спора школьников, только что приступивших к изучению электричества. Генератор, однажды приведенный в движение, питает током электродвигатель (рис. 70). Оси генератора и электродвигателя соединены между собой ременной передачей, поэтому электродвигатель поддерживает движение якоря генератора. Можно ли установку назвать вечным двигателем? К генератору по желанию можно подключить осветительную сеть или электродвигатель соединить со станком.
Решите задачу:
«Мощность генератора 4 кВт. Какой мощности электродвигатель можно соединить с этим генератором, если КПД генератора 80% и КПД двигателя тоже 80%?»
7. Радиометр. В баллон с разреженным воздухом помещена вертушка из легких алюминиевых крылышек. С одной стороны крылышки покрыты черной краской (рис. 71). Если недалеко от этого баллона поставить источник яркого света или сильно нагретый предмет, то вертушка начнет вращаться. Можно ил радиометр назвать вечным двигателем?
8. Спинтарископ. Дно короткой трубки представляет экран, покрытый сульфидом цинка — веществом, светящимся от удара альфа-частиц (рис. 72). Близ экрана укреплена игла, на острие которой находится небольшое количество вещества, испускающего альфа-частицы. Наблюдатель, смотрящий через окуляр (при полной темноте), увидит на экране частые вспышки. Ни днем, ни ночью не прекращается эффектное зрелище, напоминающее салют в небе. Бесспорно, что описанное явление сопровождается выделением энергии. Но можно ли назвать спинтарископ вечным двигателем?
Расчёт систем — вечный двигатель
АльтИнфоЮг
Альтернативная энергетика и информация
- Главная
- Расчёты
- Расчёт идей
- Расчёт систем — вечный двигатель
ЗДОРОВЬЕ И ДЕНЬГИ ЗДЕСЬ
Чтоб вы все были здоровы и богаты долгие годы. ЖМИ!
На форумах по альтернативной энергетике, в литературе, а иногда и в патентах на изобретения, встречаются системы для получения и преобразования энергии. Иногда это замкнутые системы для получения и преобразования сверх единичной энергии, работающие по принципу получения свободной энергии или вечного двигателя, а иногда разомкнутые системы преобразования энергии. Предлагаемый метод оценки подобных энергосистем, поможет сделать энергетические и экономические расчёты различных вариантов вечных двигателей и их возможностей.
На форумах по альтернативной энергетике, в литературе, а иногда и в патентах на изобретения, встречаются системы для получения и преобразования энергии. Иногда это замкнутые системы для получения и преобразования сверх единичной энергии, работающие по принципу получения свободной энергии или вечного двигателя, а иногда разомкнутые системы преобразования энергии. Предлагаемый метод оценки подобных энергосистем, поможет сделать энергетические и экономические расчёты различных вариантов вечных двигателей и их возможностей
Как пример такой системы можно рассматривать вариант двигатель – генератор. Здесь предполагается, что электродвигатель вращает электрогенератор. Часть вырабатываемой генератором энергии идёт на поддержание вращения электродвигателя, а часть может использоваться потребителями.
В другом варианте система содержит электрогенератор, энергия от которого идёт на нагрев парового котла. Пар вращает турбину, от которой работает генератор. Часть оставшейся электрической энергии должна использоваться потребителями. Это варианты представляют собой замкнутые системы, в которых энергия не подводится извне.
Вариантом подобных систем можно считать и незамкнутые энергосистемы. В них энергия подводится извне, а методом различных преобразований и перераспределений энергии, предполагается получать энергии больше, чем затратили. Такие системы нередко патентуются как изобретения. Читатели сайта также иногда предлагают для рассмотрения и анализа различные варианты похожих систем, публикуемых в интернете. Сделать технические и экономические расчёты подобных систем обычными методами довольно сложно. Применим предлагаемый способ расчёта для анализа такой системы, по патенту № 2275527 «Морской энергетический комплекс» который разработан и запатентован Общевойсковой академией Вооруженных Сил Российской Федерации. Если коротко, то энергетический комплекс представляет собой плавающую платформу, на которой установлено несколько источников альтернативной энергии, таких как ветродвигатели, солнечные батареи, волновые электростанции, термоэлементы, работающие на разнице температур. Электростанция на термоэлементах, хоть и часто упоминается, но очень мало подробно описывается и анализируется в литературе, поэтому заслуживает отдельной статьи. От этих источников энергии работает электролизёр для получения водорода. На этом водороде работает дизельный генератор, который должен быть основным источником электрической энергии для нескольких других потребителей, в том числе и для нагрева батарей термоэлементов. Эта система большей частью не замкнута, а последовательное преобразование энергии по замыслу авторов, предназначено для повышения энергетической эффективности.
Традиционный расчёт эффективности этой и подобных систем энергообеспечения очень сложный. Нужно сконструировать все элементы системы, рассчитать их режимы работы для обеспечения заданной производительности. Поэтому желательно первоначально сделать предварительный упрощённый расчёт, который займёт совсем немного времени и позволит сэкономить на работе инженеров-конструкторов. Задача облегчается тем, что цепочка по преобразованию энергии состоит из известных элементов и с достаточно хорошо известными характеристиками по к. п.д. каждого элемента, который и возьмём за основу в расчётах.
И, так, мы получили некоторое количество энергии от имеющихся источников (ветродвигатели, солнечные элементы, термоэлементы, волновые электростанции). Далее полученная энергия подаётся на электролизёр. При обычном электролизе расход электроэнергии на получение кубометра водорода составляет 5,1-5,6 кВт.ч электроэнергии. При использовании щелочного метода под давлением, расход электроэнергии составит 4,3-4,7 кВт.ч на кубометр водорода. Если использовать самый экономичный способ электролиза с твёрдым электролитом, то расход электроэнергии для получения кубометра водорода, составит 3,5 кВт.ч., а достижимый к.п.д. электролиза может составлять 90%.
Полученный водород частично сжимается для резервного хранения, а частично подаётся на основной источник энергии – дизельный генератор, приспособленный для работы на водороде. К.п.д. такого дизельного двигателя принимаем за 30%, а к.п.д. электрогенератора 80%. Для рассматриваемой части энергетического комплекса, как и для любой другой системы с последовательным преобразованием энергии, общий к. п.д. равен произведению к.п.д. каждого элемента системы.
Тогда получаем к.п.д. системы:
К.П.Д. = 0,9 • 0,3 • 0,8 = 0,216 = 21,6%.
Таким образом, после преобразования электричества в электричество, мы теряем 78,4% первоначально полученной дорогими способами электрической энергии. Точность подобных расчётов зависит от того, насколько точно брали цифры по к.п.д. каждого элемента и все ли потери учитывали. Конкретно в этом расчёте мы не учитывали расход энергии на собственные нужды энергетического комплекса, расход энергии на компрессор для сжатия водорода, а возможно и другие. Поэтому реальные результаты могут немного отличаться от полученных при этом расчёте, но погрешность должна быть не очень большой и её можно корректировать, если уточнить характеристики конкретного оборудования.
Для закрепления материала рассчитаем кольцевую замкнутую систему, под которой имеется в виду энергосистема, в которой на вход подаётся энергия, полученная на выходе системы. В таких системах типа вечный двигатель часто предполагается отбор энергии на нужды потребителей. Например, это ранее упоминавшаяся система двигатель — генератор. При значениях к.п.д. электродвигателя 90% и к.п.д. электрогенератора 80%, общий к.п.д. системы будет равен 72%. Это значит, что для поддержания в работе системы двигатель – генератор, на электродвигатель надо подавать на 28% больше энергии, чем выдаёт электрогенератор. Отбирать от такой системы энергию невозможно.
Способ предварительного расчёта можно использовать в самых разных системах, в том числе и при преобразовании одних видов энергии в другие. При необходимости для проведения расчётов с преобразованием различных видов энергии, например, тепловой, электрической, механической, рекомендую для их сравнения и расчёта к.п.д., сделать перевод энергии или мощности в одинаковые единицы измерения, например, джоули (Дж.), ватты (Вт.) или кратные им единицы используя справочник по переводу единиц измерения.
Запчасти для двигателей Японских грузовиков
Популярные товары
В наличии
Палец поршневой E13C 04133-E0020
Палец поршневой E13C 04133-E0020
Цена 10 000 ₽
В наличии
Распредвал 6D16 ME074625 (24 клапанный)
Распредвал 6D16 ME074625 (24 клапанный)
Цена 226 600 ₽
В наличии
Шатун J08E 13260-E0100
Шатун J08E 13260-E0100
Цена 42 400 ₽
В наличии
Шатун E13C 13260-E0140
Шатун E13C 13260-E0140
Цена 24 200 ₽
В наличии
Распредвал E13C 13501-E0380
Распредвал E13C 13501-E0380
Цена 68 200 ₽
В наличии
Сайлентблок реактивной тяги ISUZU 10PD1 1-51519-113-0
Сайлентблок реактивной тяги ISUZU 10PD1 1-51519-113-0
Цена 1 600 ₽
В наличии
Поршневая 4D33 (II модель)
Поршневая 4D33 (II модель)
Цена 53 000 ₽
В наличии
Форсунка топливная N04C 095000-0760 (Евро-4)
Форсунка топливная N04C 095000-0760 (Евро-4)
Цена 45 400 ₽
В наличии
Поршневая SL MAZDA TITAN
Поршневая SL MAZDA TITAN
Цена 68 200 ₽
В наличии
Форсунка топливная J08E 095000-1170 (евро-4)
Форсунка топливная J08E 095000-1170 (евро-4)
Цена 41 600 ₽
В наличии
Поршневая 6M61 ME995551
Поршневая 6M61 ME995551
Цена 136 300 ₽
В наличии
Гильза ДВС 4HG1
Гильза ДВС 4HG1
Цена 5 300 ₽
Многие водители прекрасно знают, что именно японские грузовики являются одними из наиболее надежных и безопасных. Данный факт легко объясняется использованием производителями транспорта действительно качественных деталей, причем без исключения. Однако любая, даже самая надежная техника со временем требует ремонта. Если подошел срок, и нужно что-то заменить или починить, вы легко найдете любые запчасти для японских грузовиков в нашем интернет магазине.
Все запчасти для дизельных двигателей разделяются на две категории:
- оригинальные;
- неоригинальные
Мало кто знает, но во многих случаях подобранная с умом даже неоригинальная деталь может выполнять свои функции не хуже, чем оригинальные. Для того чтобы купить качественные запчасти, совсем не обязательно быть настоящим профессионалом. Достаточно просто выбрать хороший магазин, где знающие люди помогут вам подобрать идеальный вариант.
Чтобы найти интересующую деталь, мы специально создали удобный каталог. Кроме того, цена на нашу продукцию точно вас приятно удивит.
Запчасти для дизельных двигателей на грузовики, спецтехнику и легковые авто японского производства. Автозапчасти в наличии на складе во Владивостоке. Доставка в любой город России транспортными компаниями, отгрузка на следующий день. Оплатить заказ можно он-лайн на сайте или по счету.
Вечный двигатель — презентация онлайн
Презентацию выполнили:
Бабамурадова У.,
Гасымова С.
Вечный двигатель
Вечный двигатель (лат. Perpetuum Mobile) — воображаемое
устройство, позволяющее получать полезную работу, большую, чем
количество сообщённой ему энергии (КПД больше 100 %).
Какие существуют вечные двигатели?
Вопрос: Какие существуют вечные двигатели?
Ответ: Никакие. Но, несмотря на это, существует
классификация вечных двигателей.
Вечный двигатель (perpetuum mobile) — делится на
вечные двигатели первого рода и второго рода.
Причины, по которой их нельзя построить, называются
первое и второе начала термодинамики.
Осознание того, что создание вечного двигателя
невозможно, подвигло Парижскую академию наук в 1775
году отказать в рассмотрении всех подобных проектов
(основанием было примерно следующее: «халявы не
бывает»).
Вечный двигатель первого рода предполагал
работать без извлечения энергии из
окружающей среды.
Вечный двигатель второго рода — это машина,
которая уменьшает энергию теплового
резервуара и целиком превращает ее в работу
без каких либо изменений в окружающей среде.
Модель вечного двигателя
На рис. 1 показана одна из древнейших конструкций вечного
двигателя. Она представляет зубчатое колесо, в углублениях
которого прикреплены откидывающиеся на шарнирах грузы.
Геометрия зубьев такова, что грузы в левой части колеса всегда
оказываются ближе к оси, чем в правой. По замыслу автора, это, в
согласии с законом рычага, должно было бы приводить колесо в
постоянное вращение. При вращении грузы откидывались бы
справа и сохраняли движущее усилие.
Однако, если такое колесо изготовить, оно останется неподвижным.
Дифференциальная причина этого факта заключается в том, что
хотя справа грузы имеют более длинный рычаг, слева их больше по
количеству. В результате моменты сил справа и слева оказываются
равны.
Рис. 1. Одна из древнейших конструкций вечного
двигателя
Арабский вечный
двигатель
Индийский или арабский
вечный двигатель с
небольшими косо
закрепленными сосудами,
частично наполненными
ртутью.
Вечный двигатель на постоянных
магнитах
Вечный двигатель и закон
Архимеда
На рис. 2 показано устройство ещё одного двигателя. Автор
решил использовать для выработки энергии закон Архимеда.
Закон состоит в том, что тела, плотность которых меньше
плотности воды, стремятся всплыть на поверхность. Поэтому
автор расположил на цепи полые баки и правую половину
поместил под воду. Он полагал, что вода будет их выталкивать
на поверхность, а цепь с колёсами, таким образом, бесконечно
вращаться.
Здесь не учтено следующее: выталкивающая сила — это
разница между давлениями воды, действующими на нижнюю
и верхнюю части погруженного в воду предмета. В
конструкции, приведённой на рисунке, эта разница будет
стремиться вытолкнуть те баки, которые находятся под водой в
правой части рисунка. Но на самый нижний бак, который
затыкает собой отверстие, будет действовать лишь сила
давления на его правую поверхность. И она будет превышать
суммарную силу, действующую на остальные баки. Поэтому вся
система просто прокрутится по часовой стрелке, пока не
выльется вода.
Рис. 2. Конструкция вечного двигателя, основанного
на законе Архимеда
Некоторые примеры «вечных
двигателей»
Колесо с перекатывающимися
шарами
Идея изобретателя: Колесо с
перекатывающимися в нем тяжелыми
шариками. При любом положении колеса грузы
на правой его стороне будут находиться
дальше от центра, чем грузы на левой
половине. Поэтому правая половина должна
всегда перетягивать левую и заставлять колесо
вращаться. Значит, колесо должно вращаться
вечно.
Почему двигатель не работает: Хотя грузы на
правой стороне всегда дальше от центра, чем
грузы на левой стороне, число этих грузов
меньше ровно настолько, чтобы сумма сил
тяжестей грузов, умноженных на проекцию
радиусов, перпендикулярную к направлению
силы тяжести, справа и слева были равны
(FiLi = FjLj).
Цепочка шаров на треугольной
призме
Идея изобретателя: Через трехгранную
призму перекинута цепь из 14 одинаковых
шаров. Слева четыре шара, справа — два.
Остальные восемь шаров уравновешивают
друг друга. Следовательно, цепь придет в
вечное движение против часовой стрелки.
Почему двигатель не работает: Грузы
приводит в движение только составляющая
силы тяжести, параллельная наклонной
поверхности. На более длинной поверхности
больше грузов, но и угол наклона поверхности
пропорционально меньше. Поэтому сила
тяжести грузов справа, умноженная на синус
угла, равна силе тяжести грузов слева,
умноженной на синус другого угла.
Еще в начале XVII века замечательный нидерландский физик и инженер Симон
Стевин (1548–1620), видимо первым в истории, сделал всё наоборот.
Экспериментируя с трехгранной призмой и цепью из 14 одинаковых шаров, он
предположил, что вечный двигатель вообще невозможен (это закон природы), и
вывел из этого принципа закон равновесия сил на наклонной плоскости: силы
тяжести, действующие на грузы, пропорциональны длинам плоскостей, на
которых они лежат. Из этого принципа вырос векторный закон сложения сил и
представление о том, что силы нужно описывать новым математическим
объектом — вектором.
Кроме этого, Симон Стевин сделал много глубоких, пионерских работ в физике и
математике. Он обосновал и ввел в оборот в Европе десятичные дроби,
отрицательные корни уравнений, сформулировал условия существования корня в
данном интервале и предложил способ его приближенного вычисления. Стевин
был, наверное, первым прикладным математиком, который доводил свои
вычисления до числа. Для решения конкретных практических задач он
постоянно развивал прикладные вычисления. К ним Стевин относил и
бухгалтерию, как науку о рациональном хозяйствовании, то есть он стоял у
истоков математических методов в экономике. Стевин считал, что «цель
бухгалтерского учета — определение всего народного богатства страны». Он был
суперинтендантом по военным и финансовым вопросам у великого полководца,
создателя современной регулярной армии Морица Оранского. Его должность в
современных терминах — «заместитель командующего по тылу».
«Птичка Хоттабыча»
Идея изобретателя: Тонкая стеклянная
колбочка с горизонтальной осью посередине
впаяна в небольшую емкость. Свободным
концом колбочка почти касается ее дна. В
нижнюю часть игрушки налито немного эфира,
а верхняя, пустая, обклеена снаружи тонким
слоем ваты. Перед игрушкой ставят стаканчик
с водой и наклоняют ее, заставляя «попить».
Птичка начинает два-три раза в минуту
наклоняться и окунать головку в стаканчик. Раз
за разом, непрерывно, днем и ночью кланяется
птичка, пока в стаканчике не кончится вода.
Почему это не вечный двигатель: Голова и клюв птички
покрыты ватой. Когда птичка «пьет воду», вата пропитывается
водой. При испарении воды температура головы птички
снижается. В нижнюю часть туловища птички налит эфир, над
которым находятся пары эфира (воздух откачан). При
охлаждении головы птички давление паров в верхней части
снижается. Но давление в нижней части остается тем же.
Избыточное давление паров эфира в нижней части поднимает
жидкий эфир по трубочке вверх, голова птички тяжелеет и
наклоняется к стакану.
Как только жидкий эфир дотечет до конца трубочки, пары
теплого эфира из нижней части попадут в верхнюю, давление
паров сравняется и жидкий эфир потечет вниз, а птичка снова
поднимет клюв, при этом захватив воду из стакана. Испарение
воды начинается снова, голова охлаждается и всё повторяется.
Если бы вода не испарялась, то птичка бы и не двигалась. Для
испарения из окружающего пространства потребляется энергия
(сосредоточенная в воде и окружающем воздухе).
«Настоящий» вечный двигатель должен работать без затраты
внешней энергии. Поэтому птичка Хоттабыча в
действительности не является вечным двигателем.
Цепочка поплавков
Идея изобретателя: Высокая башня наполнена водой. Через шкивы,
установленные вверху и внизу башни, перекинут канат с 14 полыми
кубическими ящиками со стороной 1 метр. Ящики, находящиеся в воде,
под действием силы Архимеда, направленной вверх, должны
последовательно всплывать на поверхность жидкости, увлекая за
собой всю цепь, а находящиеся слева ящики спускаются вниз под
действием силы тяжести. Таким образом ящики попадают попеременно
из воздуха в жидкость и наоборот.
Почему двигатель не работает: Ящики, входящие в жидкость,
встречают весьма сильное противодействие со стороны жидкости,
причем работа на проталкивание их в жидкость не меньше работы,
совершаемой силой Архимеда при всплывании ящиков на поверхность.
Архимедов винт и водяное колесо
Идея изобретателя: Архимедов винт, вращаясь, поднимает воду
в верхний бак, откуда она вытекает из лотка струей, попадающей
на лопатки водяного колеса. Водяное колесо вращает точильный
камень и одновременно двигает, с помощью ряда зубчатых колес,
тот самый Архимедов винт, который поднимает воду в верхний
бак. Винт поворачивает колесо, а колесо — винт! Этот проект,
изобретенный еще в 1575 году итальянским механиком Страдою
Старшим, затем повторялся в многочисленных вариациях.
Почему двигатель не работает: Большая часть проектов вечных
двигателей действительно могла бы работать, если бы не
существование силы трения. Если это двигатель — должны быть
и движущиеся части, значит, недостаточно двигателю вращать
самого себя: нужно вырабатывать еще и избыточную энергию
для преодоления силы трения, которую никак не уберешь.
Машина Орфиреуса
Идея изобретателя: Некоторые изобретатели вечных
двигателей были просто жуликами, ловко надувавшими
легковерную публику. Одним из наиболее выдающихся
«изобретателей» был некий доктор Орфиреус
(настоящая фамилия — Бесслер). Основным элементом
его двигателя было большое колесо, которое будто бы
не только вращалось само собой, но и поднимало при
этом тяжелый груз на значительную высоту.
Почему двигатель не работает: «Вечный двигатель»
оказался далеко не вечным — его приводили в
действие брат Орфиреуса и служанка, дергая за
искусно спрятанный шнурок.
Магнит и желоба
Идея изобретателя: Сильный магнит помещается на
подставке. К ней прислонены два наклонных желоба,
один под другим, причем верхний желоб имеет
небольшое отверстие в своей верхней части, а нижний на
конце изогнут. Если на верхний желоб положить
небольшой железный шарик, то вследствие притяжения
магнитом он покатится вверх, однако, дойдя до отверстия,
провалится в нижний желоб, скатится по нему,
поднимется по конечному закруглению и вновь попадет
на верхний желоб. Таким образом, шарик будет бегать
непрерывно, осуществляя тем самым вечное движение.
Проект этого магнитного perpetuum mobile описал в XVII
веке английский епископ Джон Вилкенс.
Почему двигатель не работает: Устройство работало
бы, если бы магнит действовал на металлический шарик
только во время его подъема на подставку по верхнему
желобу. Но вниз шарик скатывается замедленно под
действием двух сил: тяжести и магнитного притяжения.
Поэтому к концу спуска он не приобретет скорость,
необходимую для поднятия по закруглению нижнего
желоба и начала нового цикла.
«Вечный водопровод»
Идея изобретателя: Давление воды в
большом баке должно постоянно
выжимать воду по трубе в верхнюю
емкость.
Почему двигатель не работает: Автор
проекта не понимал, что
гидростатический парадокс в том и
состоит, что уровень воды в трубе
всегда остается таким же, как в баке.
Автоматический подзавод
часов
Идея изобретателя: Основа устройства —
ртутный барометр крупных размеров: чаша с
ртутью, подвешенная в раме, и опрокинутая
над ней горлышком вниз большая колба с
ртутью. Сосуды укреплены подвижно один
относительно другого; при увеличении
атмосферного давления колба опускается и
чаша поднимается, при уменьшении же
давления — наоборот. Оба движения
заставляют вращаться небольшое зубчатое
колесо всегда в одну сторону и через систему
зубчатых колес поднимают гири часов.
Почему это не вечный двигатель:
Необходимая для работы часов энергия
«черпается» из окружающей среды. По сути это
мало чем отличается от ветряного двигателя —
разве что исключительно малой мощностью.
Масло, поднимающееся по фитилям
Идея изобретателя: Жидкость, налитая в нижний
сосуд, поднимается фитилями в верхний сосуд,
имеющий желоб для стока жидкости. По стоку
жидкость падает на лопатки колеса, приводя его во
вращение. Далее стекшее вниз масло снова
поднимается по фитилям до верхнего сосуда. Таким
образом, струя масла, стекающая по желобу на
колесо, ни на секунду не прерывается, и колесо
вечно должно находиться в движении.
Почему двигатель не работает: С верхней,
загнутой части фитиля жидкость стекать вниз не
будет. Капиллярное притяжение, преодолев силу
тяжести, подняло жидкость вверх по фитилю — но
ведь та же причина удерживает жидкость в порах
намокшего фитиля, не давая ей капать с него.
Колесо с откидывающимися
грузами
Идея изобретателя: Идея основана на
применении колеса с неуравновешенными
грузами. К краям колеса прикреплены
откидные палочки с грузами на концах. При
всяком положении колеса грузы на правой
стороне будут откинуты дальше от центра,
нежели на левой; эта половина,
следовательно, должна перетягивать левую и
тем самым заставлять колесо вращаться.
Значит, колесо будет вращаться вечно, по
крайней мере, до тех пор, пока не перетрется
ось.
Почему двигатель не работает: Грузы на
правой стороне всегда дальше от центра,
однако неизбежно такое положение колеса,
при котором число этих грузов меньше, чем на
левой. Тогда система уравновешивается —
следовательно, колесо не будет вращаться, а,
сделав несколько качаний, остановится.
Установка инженера Потапова
Идея изобретателя: Гидродинамическая тепловая
установка Потапова с КПД, превышающим 400%.
Электродвигатель (ЭД) приводит в движение насос
(НС), заставляющий циркулировать воду по контуру
(показано стрелками). Контур содержит
цилиндрическую колонку (ОК) и батарею отопления
(БТ). Окончание трубы 3 можно подключить к колонке
(ОК) двумя способами: 1) к центру колонки; 2) по
касательной к окружности, образующей стенку
цилиндрической колонки. При подключении по
способу 1 количество тепла, отдаваемое воде, равно (с
учетом потерь) количеству тепла, излучаемому
батареей (БТ) в окружающее пространство. Но как
только происходит подключение трубы по способу 2,
количество излучаемого батареей (БТ) тепла
увеличивается в 4 раза! Измерения, проведенные
нашими и зарубежными специалистами, показали, что
при подводе 1 кВт к электродвигателю (ЭД) батарея
(БТ) дает столько тепла, сколько должно было бы
получаться при затрате 4 кВт. При подключении трубы
по способу 2 вода в колонке (ОК) получает
вращательное движение, и именно этот процесс
приводит к увеличению количества отдаваемого
батареей (БТ) тепла.
Почему двигатель не работает:
Описанная установка действительно
была собрана в НПО «Энергия» и, по
утверждению авторов, работала.
Изобретатели не ставили под сомнение
правильность закона сохранения
энергии, но утверждали, что двигатель
черпает энергию из «физического
вакуума». Что невозможно, т. к.
физический вакуум имеет самый низкий
из возможных уровней энергии и черпать
из него энергию нельзя.
•Наиболее вероятным
представляется более
прозаическое объяснение: имеет
место неравномерный нагрев
жидкости по сечению трубы и изза этого возникают ошибки в
измерении температуры. Не
исключено также, что энергия
помимо воли изобретателей
«закачивается» в установку из
электрической цепи.
Луна и планеты
Идея изобретателя: Вечное движение
Луны вокруг Земли и планет вокруг
Солнца.
Почему двигатель не работает: Здесь
налицо смешение понятий: «вечный
двигатель» и «вечное движение».
Полная (потенциальная и кинетическая)
энергия Солнечной системы есть
величина постоянная, и если мы
захотим за ее счет совершить работу
(что, в принципе, не исключено), то эта
энергия будет уменьшаться. Но вот
«бесплатной» работы мы всё равно не
получим.
И все-таки он существует?
Французская академия
наук, отказавшаяся когдато принимать на
рассмотрение проекты
вечных двигателей, тем
самым притормозила
технический прогресс,
надолго задержав
появление целого класса
удивительных механизмов
и технологий. Лишь
немногие разработки
сумели пробить себе дорогу
сквозь этот заслон.
ВЕЧНЫЙ ДВИГАТЕЛЬ В ЧАСАХ
Одна из них — не требующие завода часы,
которые по иронии судьбы сегодня
выпускаются именно во Франции.
Источником энергии служат колебания
температуры воздуха и атмосферного
давления в течение дня. Специальная
герметическая емкость в зависимости
от изменения среды слегка «дышит».
Эти движения передаются на ходовую
пружину, подзаводя ее. Механизм
продуман так тонко, что изменение
температуры всего на один градус
обеспечивает ход часов в течение двух
последующих суток. При условии
исправности этот механизм будет
функционировать ровно столько,
сколько светит Солнце и существует
Земля, то есть практически вечно.
Патенты и авторские свидетельства на
вечный двигатель
В Российской Федерации заявки
на патентование вечного
двигателя не рассматриваются
Действительно ли вечный двигатель — это просто? | Проза жизни
Строго говоря, писать «вечный» двигатель без кавычек, как мы это сделали в предыдущей статье, неверно, так как всякая генерирующая установка имеет изнашивающиеся детали, требующие своевременной замены, а также технического обслуживания. В данной статье мы приведем пример модели генератора энергии, не требующего для своей работы наличия гравитационной аномалии. Вы сможете убедиться, что «вечный» двигатель — это просто.
Из школьного курса физики (6-й класс) известен «вечный» двигатель с поплавками.
Конструкция вечного двигателя, основанного на законе Архимеда
Фото: Предположительно Dims, ru. wikipedia.org
Он состоит из замкнутой цепи поплавков, находящихся в емкости, разделенной пополам вертикально. В одну половину емкости залита жидкость, во второй находится воздух. В верхней части емкости поплавки свободно переходят из одной половины на другую через ролик, в нижней — расположен специальный клапан, не дающий воде переливаться и восстановить равномерный уровень жидкости в емкости по закону сообщающихся сосудов.
Предположим, что в качестве жидкости используется вода, то есть широко распространенный ресурс. Итак, с одной стороны емкости на поплавки действует выталкивающая сила Архимеда, а с другой, поскольку воды нет, не действует. Гипотетически налицо разность сил. К одному из роликов (любому) через шток крепим электродвигатель и получаем автономный генератор энергии. Мощность генератора определяется размерами установки (большие поплавки) и высотой емкости (чем выше, тем больше выталкивающая сила).
Эта модель не работает по одной причине. Не создан пока еще такой клапан, который бы пропускал поплавки, но не пропускал воду. А поскольку клапана нет — вода переливается из одной части емкости в другую, уровень жидкости выравнивается, движение поплавков (и получение электрической энергии) прекращается.
На практике этот классический, можно сказать, хрестоматийный образец не работающего «вечного» двигателя — работает. Далее мы покажем, каким образом это возможно.
Коэффициент полезного действия (КПД) рассчитывается по формуле:
- КПД = Ап / Аз, где КПД — это отношение полезной и затраченной энергии.
Для «вечного» двигателя КПД стремится к 100%. К примеру, для нас КПД Солнца 100%.
Закон сохранения энергии выглядит как сумма потенциальной и кинетической энергии.
- Еп + Ек = 0
В данном случае Еп — это сила гравитационного поля, действующего на поплавки.
Нетрудно видеть, что для работы поплавкового «вечного» двигателя необходима хотя бы минимальная разница уровней жидкости в двух отделениях емкости.
Фото: Depositphotos
Этого можно добиться путем дополнения к конструкции еще одной потенциальной силы, а именно постоянного магнитного поля. Магнитное поле очень напоминает гравитационное поле. Оно действует постоянно, не затрачивая энергию извне и без видимого уменьшения силы действия. И его также можно отнести к источникам потенциальной энергии.
Закон сохранения энергии для «вечного» двигателя будет выглядеть уже так:
- Еп = Е гравитационного поля + Е магнитного поля
(Ег + Ем) + Ек = 0
То есть с теоретической точки зрения нарушения закона сохранения энергии нет.
Практически остается залить в емкость вместо воды жидкость, обладающую магнитными свойствами, и разместить над одной стороной емкости постоянный магнит. Жидкость притянется к магниту, нарушив тем самым равновесие в емкости. И не работающий «вечный» двигатель заработает. Как вы понимаете, клапан внизу емкости становится вовсе ненужным.
Следующий естественно возникающий вопрос: где взять жидкость, обладающую магнитными свойствами («магнитную жидкость»)? Технические решения этой проблемы могут быть различными.
В заключение ответим на неочевидный, но важный вопрос: как остановить «вечный» двигатель? Очень просто: снять с емкости магнит.
Теги:
закон сохранения энергии,
вечный двигатель,
магнитное поле
Возможен ли двигатель с постоянными магнитами?
Недавно мы опубликовали статью о силовой электронике с заголовком «Уникальный двигатель использует только постоянные магниты — электроэнергия не требуется». Мы получили шквал критики за то, что это звучит как вечный двигатель и противоречит закону сохранения энергии и закону термодинамики. Некоторые инженеры сказали, что это должно было быть датировано 1 апреля, потому что это, должно быть, шутка. Меня заставили поверить, что такой мотор существует, но мотора не существует — по крайней мере, пока.
Выяснилось, что заголовок был неверным. В нем должно было быть сказано: «Новое открытие может привести к коммерческому производству двигателей с постоянными магнитами». Соавтор оригинальной статьи, доктор Кеннет Козека, открыл способ использования постоянных магнитов для создания механического движения. В статье должно было быть ясно, что это открытие может привести к двигателю с постоянными магнитами, но пока нет. Затем мы попросили доктора Козеку объяснить предысторию своего открытия, и он предоставил то, что вы увидите в тексте ниже. Прочитав это объяснение, вы сможете решить, считаете ли вы этот подход осуществимым.
Доктор Козека говорит, что легко представить себе силу притяжения между двумя магнитами, выполняющими работу за нас, например, вращение двигателя, когда они сближаются. Проблема, конечно, в том, что энергия должна быть потрачена на разрыв магнитов, если мы хотим, чтобы они снова работали на нас. Таким образом, нет никакой пользы в том, чтобы магниты работали на нас.
Ученые и изобретатели пытались использовать только постоянные магниты для привода двигателя. Другие отвергли идею двигателя, приводимого в движение только постоянными магнитами, как противоречащего законам термодинамики. Мы не понимали источник электромагнитной энергии, ответственный за магнитные силы. Уже в 1926, квантовая физика описала собственный спин или угловой момент неспаренного электрона в ферромагнитном материале как источник. Доктор Фейнман (лауреат Нобелевской премии по физике) описывает вращение как «вечное» в своих лекциях об электромагнитной энергии.
В физике существует несколько теорий, предлагающих источник электромагнитной энергии, переносимой потоками виртуальных фотонов, исходящих от атомного электрона. Каким бы ни был источник, он присущ и изобилен. Таким образом, идея двигателя, приводимого в движение одними только постоянными магнитами, вполне осуществима, и ее нельзя отвергать как нарушение закона сохранения энергии.
Двигатель с постоянными магнитами не будет производить энергию и не будет вечным двигателем. Вместо этого он просто использовал бы электромагнитную энергию, передаваемую угловым моментом электрона в виде магнитных сил. Хотя квантовая физика описывает угловой момент как источник энергии, некоторые ученые и неспециалисты придерживаются неверной парадигмы, согласно которой двигатели с постоянными магнитами противоречат основным законам физики.
Представленное здесь открытие проливает свет на очень необычное явление. Два постоянных магнита с противоположными полюсами, обращенными друг к другу, способны создавать экваториальное притяжение и полярное отталкивание без изменения полярности магнита и без использования другого источника энергии. Противоположные полюса не вызывают отталкивания. Отталкивание также не является побочным продуктом инерции или импульса горизонтального притяжения. Это ясно видно из демонстрации полярного отталкивания, происходящего без предварительного создания горизонтального притяжения. Посетите сайт www.kedronenergy.com, чтобы посмотреть видеоролики, демонстрирующие отдельно горизонтальное притяжение и вертикальное отталкивание.
Удивительно, если не захватывает дух, наблюдать, как два постоянных магнита с обращенными противоположными полюсами создают одновременно экваториальное (горизонтальное) притяжение и полярное (вертикальное) отталкивание, которые можно использовать для создания последовательности притяжения и отталкивания. Таким образом, не нужно тратить энергию на то, чтобы разъединить магниты после того, как они притянутся и сделают работу за нас. Вместо этого магниты отделяются друг от друга. И фаза притяжения, и фаза отталкивания могут выполнять для нас работу, например, управлять электрическим генератором. Есть два рабочих такта по сравнению с одним рабочим тактом в двигателе внутреннего сгорания. Это удивительное открытие нельзя ни сбросить со счетов, ни опровергнуть, потому что оно легко воспроизводимо кем угодно. На сайте Kedron представлено видео, демонстрирующее это явление с помощью небольшого аппарата. Инструкции предоставляются для воспроизведения этого небольшого устройства. Редко, если вообще когда-либо, такое важное научное открытие так легко проверяется.
Нетрудно представить, как можно использовать рабочий ход двигателя внутреннего сгорания для создания непрерывного движения. Точно так же легко представить, как притяжение и отталкивание силовых ударов постоянных магнитов можно использовать для создания непрерывного движения, аналогичного конструкции обычного электродвигателя или двигателя внутреннего сгорания. На веб-сайте Kedron доступно видео, в котором показано, как несколько пар магнитов, соединенных вместе, могут создавать непрерывное движение, используя магниты на обоих концах своего пути. Также предоставляется моделирование того, как несколько пар постоянных магнитов могут быть соединены для создания непрерывного движения. Это, пожалуй, самая простая версия машины или «мотора», создающего непрерывное движение. Лучшие конструкции будут использоваться в коммерческом подразделении.
Kedron ищет партнерство с компаниями, которые могут разрабатывать и производить коммерческие двигатели с постоянными магнитами. Это открытие было опубликовано для поиска поддержки для дальнейшего развития и внедрения. Подробная информация о конструкции двигателя станет доступна после получения патентной защиты и коммерческого производства компаниями-производителями.
Патенты на вечные двигатели и заявки на патенты (класс 415/916)
Патенты на вечные двигатели (класс 415/9)16)
СИСТЕМА ПРОИЗВОДСТВА ЭЛЕКТРОЭНЕРГИИ НА ВОЗДУХЕ
Номер публикации: 20150137521
Резюме: Система пневматической турбины вырабатывает электроэнергию, используя входную электрическую энергию для создания постоянного потока воздуха, который сжимается в пневматическую энергию, которая преобразуется в механическую энергию для производства электрической энергии, так что общая энергия выходная мощность, возникающая в результате объединенных сил ветра, пневматической, электрической и механической энергии, больше, чем потребляемая электрическая энергия. Камера многократного сжатия, состоящая из стартера и турбины воздухозаборника, всасывает воздух в корпус и сжимает воздух. В коридоре реактивного движения воздух дополнительно сжимается с помощью форсунок, где воздух подается в коридор турбин, вырабатывающих электроэнергию, с микрокомпрессионными турбинами, установленными на опоре, которая подключена к стабилизирующему двигателю и электрическому генератору. Турбины микрокомпрессии дополнительно сжимают воздух и передают механическую энергию генератору. Корпус перенаправляет лишний воздух обратно в систему.
Тип:
ЗаявкаПодано:
18 ноября 2014 г.Дата публикации:
21 мая 2015 г.Изобретатель:
Габриэль Огайочиоя Обадан
УСТРОЙСТВО ПРЕОБРАЗОВАНИЯ ЭНЕРГИИ НА ОСНОВЕ ГРАВИТАЦИИ
Номер публикации: 20150130192
Резюме: Настоящее изобретение предлагает систему преобразования энергии на основе гравитации. Система преобразования энергии на основе гравитации включает в себя круговой шкив, первый контейнер, второй контейнер, блок преобразования энергии вращения и блок выработки электроэнергии. Первый контейнер и второй контейнер подвешены по обеим сторонам круглого элемента с помощью соединительного элемента. В соответствии с настоящим изобретением вес первого контейнера и второго контейнера изменяется для получения непрерывных восходящих и нисходящих движений первого контейнера и второго контейнера. Восходящие и нисходящие движения первого контейнера и второго контейнера создают двунаправленное круговое движение на круглом элементе. Блок преобразования энергии вращения преобразует двунаправленное круговое движение круглого элемента в однонаправленное вращательное движение.
Тип:
ЗаявкаПодано:
17 мая 2013 г.Дата публикации:
14 мая 2015 г.Изобретатель:
Логанатан Палани
Глубоководная гидроэлектростанция
Номер патента: 8987932
Реферат: Глубоководная энергетическая система включает в себя первоначально эвакуированное ограждение, имеющее стенки соответствующей прочности или усиления для сохранения его конструктивной целостности при глубоководном давлении; силовая ось, проходящая через ограждение от северного полюса ниже южного полюса ограждения; предпочтительно вогнутые лопатки турбины, закрепленные на опорной раме, прикрепленной к силовой оси в продольной плоскости корпуса; и впускные отверстия внутри кожуха, расположенные в плоскости лопастей турбины по широте и получающие приток окружающей глубокой воды против лопастей, в которых парное силовое воздействие от потока жидкости вызывает вращение лопастей и приводного моста. крепится к раме. Тяговая платформа жестко прикреплена внутри кожуха к приводному мосту над турбиной и к генератору, закрепленному на упорной платформе.
Тип:
ГрантПодано:
9 июня 2014 г.Дата патента:
24 марта 2015 г.Изобретатели:
Лакдас Нанаяккара, Правин Нанаяккара
Кинетическая турбинная система с магнитным приводом
Номер патента: 8967947
Аннотация: Система принимает внешнюю кинетическую энергию в различных формах, таких как ветер, дождь или водопад, и производит электричество с помощью традиционных катушек и магнитов или других средств. Во время перерывов в получении внешней кинетической энергии ветроколесо, водяное колесо или система рычагов замедляются и в конце концов останавливаются. Система магнитов, установленных на системе рычагов и окружающем стационарном корпусе 800, способствует вращению системы рычагов, так что системе рычагов потребуется дополнительное время для остановки после затишья внешней кинетической энергии. Искусное размещение и ориентация магнитов 250 стационарного корпуса по отношению к магнитам 200 подвижных рычагов способствует вращению системы рычагов, приводимой в действие внешними кинетическими силами. Углы стационарных магнитов корпуса заставляют магниты рычага двигаться в нужном направлении 900 вращения. Система повышает механическую эффективность.
Тип:
ГрантПодано:
13 декабря 2011 г.Дата выдачи патента:
3 марта 2015 г.Изобретатель:
Леланд Х. Хасс
УСТРОЙСТВО ДЛЯ ПРОИЗВОДСТВА ЭЛЕКТРОЭНЕРГИИ ИЗ СИСТЕМЫ ЦИРКУЛЯЦИИ ВОДЫ ПОД НАПОРОМ
Номер публикации: 20150001141
Реферат: Раскрыто устройство для выработки электроэнергии из системы циркуляции воды под давлением. В устройстве используется поток воды внутри трубчатого элемента для создания механической силы для вращения ротора. Электрический генератор включает в себя ротор, содержащий крыльчатку, при этом ротор сконфигурирован для приема потока жидкости внутри якоря с электромагнитной индукцией из трубчатого элемента, статор сконфигурирован для выработки электрической энергии внутри множества катушек с использованием магнитного потока, генерируемого электромагнитной индукцией. якорь при вращении рядом со статором, и перепускной трубчатый элемент, выполненный с возможностью избирательного направления жидкости вокруг электрического генератора для регулирования напряжения генерируемой электрической энергии. Устройство работает в системе циркуляции воды в бассейне и подает электроэнергию на насос бассейна.
Тип:
ЗаявкаПодано:
13 мая 2014 г.Дата публикации:
1 января 2015 г.Изобретатель:
Майкл Виланд
Волновая система генерации электроэнергии
Номер патента: 8922044
Реферат: Система выработки электроэнергии с волновым приводом, в которой используется мотор-редуктор для поворота коромысла в движущихся волнообразующих бочках попеременно вверх и вниз, вызывая образование водяных волн в водоемах так, что плавучие доски в водохранилищах перемещаются создаваемыми волнами воды для вращения коробки передач, которая соединена с энергоблоком через инерционную колесную пару, и, таким образом, энергоблок приводится в действие для выработки электроэнергии.
Тип:
ГрантПодано:
26 марта 2014 г.Дата выдачи патента:
30 декабря 2014 г.Изобретатель:
Шэн-По Пэн
Аппарат для выработки электроэнергии с гидроприводом
Номер публикации: 20140375059
Реферат: Гидравлический генератор энергии представляет собой генератор энергии в виде труб с водяными колесами или шариками с вращающимися по окружности зубцами. Водяные колеса или шары вращаются потоком жидкости внутри труб. Каждое из водяных колес или каждый шар имеет ось колеса или ось шарового колеса, которая окружена металлической катушкой. Каждая металлическая катушка вращается потоком жидкости в магнитном поле, создаваемом электромагнитным слоем, прикрепленным к каждой из труб. Таким образом, благодаря вращению металлической катушки в магнитном поле электричество создается и сохраняется в устройстве накопления энергии. Другой вариант настоящего изобретения представляет собой электроэнергетическое устройство с замкнутой системой, работающее от жидкости, с ртутью в качестве движущей жидкости. Настоящее изобретение может быть использовано в водопроводных, погружных, спринклерных системах, а также в плавсредствах, таких как корабли, подводные лодки, водные мотоциклы и т. д.
Тип:
ЗаявкаПодано:
10 сентября 2014 г.Дата публикации:
25 декабря 2014 г.Изобретатель:
Кайли Ирэн Летанг
Волновая система генерации электроэнергии
Номер патента: 8
- 4
Реферат: Система выработки электроэнергии с волновым приводом, в которой используется мотор-редуктор для поворота коромысла в движущихся волнообразующих бочках попеременно вверх и вниз, вызывая образование водяных волн в водохранилище. таким образом, поплавковая доска в водоеме перемещается создаваемыми водными волнами для вращения передаточных механизмов, которые соединены с соответствующими электрогенераторами через инерционные колесные пары, и, таким образом, электрогенераторы приводятся в действие для выработки электроэнергии.
Тип:
ГрантПодано:
22 апреля 2014 г.Дата выдачи патента:
9 декабря 2014 г.Изобретатели:
Шэн-По Пэн, Шэн-Да Пэн, Кевин Пэн
ГЕНЕРАТОР ЭНЕРГИИ
Номер публикации: 20140312623
Реферат: Раскрытая силовая установка представляет собой двигатель, полезность которого заключается в выработке энергии за счет использования естественных перепадов давления, встречающихся в различных жидкостях и газах, таких как вода и газы, но не ограничиваясь ими. воздуха. Обычно это двухтактный двигатель с поршневым циклом, в котором действия поршней выполняют работу или пополняют рабочую жидкость от нижнего напора к более высокому напору, и его можно использовать, например, для выработки энергии, перекачивания жидкостей или выполнения работы.
Тип:
ЗаявкаПодано:
18 декабря 2013 г.Дата публикации:
23 октября 2014 г.Изобретатель:
Марк Р. Анто
ДВИГАТЕЛЬ ШПИНДЕЛЯ И УСТРОЙСТВО ПРИВОДА ЗАПИСЫВАЮЩЕГО ДИСКА, ВКЛЮЧАЯ ЕГО
Номер публикации: 20140285921
Реферат: Предусмотрен шпиндельный двигатель, включающий: сердечник статора, неподвижно установленный на статоре; приводной магнит, центр которого совмещен с центром сердечника статора в осевом направлении; и втулку ротора, включающую в себя корпус втулки, имеющий форму диска, часть крепления магнита, выступающую от края корпуса втулки в осевом направлении вниз и имеющую приводной магнит, установленный на ее внутренней периферийной поверхности, и часть, поддерживающую диск, выступающую из часть крепления магнита в радиальном направлении, при этом статор включает в себя базовый элемент, а тянущий магнит установлен по меньшей мере на одной из нижней поверхности опорной части диска и верхней поверхности базового элемента, обращенной к нижней поверхности опорной части диска.
Тип:
ЗаявкаПодано:
23 июля 2013 г.Дата публикации:
25 сентября 2014 г.Заявитель:
SAMSUNG ELECTRO-MECHANICS CO., LTD.Изобретатели:
Сан Гю ЛИ, Джин Сок Чан, Хон Чжу Ли
Волновая система генерации электроэнергии
Номер публикации: 20140239642
Реферат: Система выработки электроэнергии с волновым приводом, в которой используется мотор-редуктор для поворота коромысла в движущихся волнообразующих бочках попеременно вверх и вниз, вызывая образование водяных волн в водохранилище. таким образом, поплавковая доска в водоеме перемещается создаваемыми водными волнами для вращения передаточных механизмов, которые соединены с соответствующими электрогенераторами через инерционные колесные пары, и, таким образом, электрогенераторы приводятся в действие для выработки электроэнергии.
Тип:
ЗаявкаПодано:
22 апреля 2014 г.Дата публикации:
28 августа 2014 г.Изобретатели:
Шэн-По Пэн, Шэн-Да Пэн, Кевин Пэн
Волновая система генерации электроэнергии
Номер публикации: 20140217736
Реферат: Система выработки электроэнергии с волновым приводом, в которой используется мотор-редуктор для поворота коромысла в движущихся волнообразующих бочках попеременно вверх и вниз, вызывая образование водяных волн в водоемах таким образом, что плавучие доски в водохранилищах перемещаются создаваемыми волнами воды для вращения коробки передач, которая соединена с энергоблоком через инерционную колесную пару, и, таким образом, энергоблок приводится в действие для выработки электроэнергии.
Тип:
ЗаявкаПодано:
26 марта 2014 г.Дата публикации:
7 августа 2014 г.Изобретатель:
Шэн-По Пэн
Электрический генератор и способ производства электроэнергии
Номер публикации: 20140203567
Abstract: Экологически безопасный и эффективный электрический генератор и система для и способ производства электроэнергии включают источник энергии с одной или несколькими батареями, электродвигатель, работающий от батарей, гидравлический насос. приводимый в действие двигателем для повышения давления жидкости, первый гидравлический двигатель, приводимый в действие насосом, вращающийся вал, прикрепленный к первому гидравлическому двигателю, гидравлическую систему с воздушным давлением и выходной генератор переменного тока, соединенный с валом для выработки электроэнергии. В предпочтительном варианте осуществления пневматическая гидравлическая система содержит компрессор, оперативно соединенный с валом для нагнетания воздуха, механизм усиления воздуха для увеличения расхода сжатого воздуха, нагнетательный бак для повышения давления сжатого воздуха, гидравлический блок питания для повышения давления жидкости с помощью сжатого воздуха, второй гидравлический двигатель, работающий от жидкости под давлением, и генератор переменного тока для подзарядки аккумуляторов.
Тип:
ЗаявкаПодано:
21 января 2013 г.Дата публикации:
24 июля 2014 г.Изобретатель:
Мануэль Лемус
Альтернативный источник энергии
Номер публикации: 20140183865
Резюме: Настоящее изобретение может быть использовано для выработки электроэнергии с использованием изменения давления воздуха, вызванного разницей в высоте. Воздушный поток создается в трубе благодаря ее расположению, имеющему отверстия на разной высоте над уровнем моря. Нижний конец трубы находится в зоне повышенного давления, а верхний — в зоне пониженного давления. Создаваемый внутри трубы поток воздуха перемещается из зоны более высокого давления в зону более низкого давления, заставляя расположенный внутри трубы пропеллер, соединенный с электрическим генератором, вращаться и генерировать электрический ток.
Тип:
ЗаявкаПодано:
28 декабря 2012 г.Дата публикации:
3 июля 2014 г.Изобретатель:
Владимир Пучков
МЕТОД И УСТРОЙСТВО ПОЛУЧЕНИЯ ЭНЕРГИИ ИЗ АТМОСФЕРНОГО ДАВЛЕНИЯ И ВАКУУМА
Номер публикации: 20140175804
Abstract: Способ и устройство, использующие атмосферное давление и силу вакуума для выработки электроэнергии путем проталкивания воды из резервуара для воды по восходящему трубопроводу в первый вакуумный резервуар и пропускания воды вниз для удара против лопастей гидравлического генератора, установленного во втором вакуумном баке для выработки электроэнергии. Промежуточные вакуумные резервуары могут быть добавлены между резервуаром для воды и первым вакуумным резервуаром, чтобы поднять высоту первого вакуумного резервуара, чтобы увеличить воздействие стекающей вниз воды. На применяемый метод и устройство не влияет местный климат. Устройство можно установить на заднем дворе дома или на заводе, работающем круглосуточно в любое время года. Устройство может быть использовано для подачи воды на большую высоту для распределения воды.
Тип:
ЗаявкаПодано:
13 мая 2013 г.Дата публикации:
26 июня 2014 г.Изобретатель:
ВЕН-ШОУ ОУ
Глубоководная гидроэлектростанция
Номер патента: 8749086
Реферат: Глубоководная энергетическая система включает в себя первоначально эвакуированную сферу, имеющую стенки соответствующей прочности или усиление для сохранения ее структурной целостности при глубоководном давлении; силовая ось, проходящая от ее северного полюса до южного полюса сферы; лопатки турбины закреплены на опорной раме, закрепленной на оси в широтной плоскости сферы; и впускные отверстия, расположенные в плоскости лопастей турбины и принимающие приток окружающей глубокой воды против лопасти. Поток жидкости приводит во вращение силовую ось лопастей, закрепленную на раме. Упорная платформа жестко закреплена внутри сферы на силовом мосту и над турбиной, а генератор закреплен на опорной платформе на южном полюсе сферы, спирально вытесняющей воду из южного полюса системы.
Тип:
ГрантПодано:
24 января 2013 г.Дата патента:
10 июня 2014 г.Изобретатели:
Лакдас Нанаяккара, Правин Нанаяккара
НЕПРЕРЫВНАЯ СИСТЕМА ПОТОКА ВОДЫ С ЭЛЕКТРОПИТАНИЕМ / ИЛИ C.E.G.F.S.
Номер публикации: 20140103659
Резюме: Усовершенствованный автономный электрический блок, состоящий из трех отдельных компонентов, производящих электричество.
Тип:
ПриложениеФайл:
26 июня 2012 г.Дата публикации:
17 апреля 2014 г.Изобретатель:
ЭРНЕСТ ПРИОВОЛОС
ГИДРОСТАТИЧЕСКИЙ ДВИГАТЕЛЬ И СПОСОБ РАБОТЫ ГИДРОСТАТИЧЕСКОГО ДВИГАТЕЛЯ
Номер публикации: 20140079531
Реферат: Предложен двигатель, использующий гидростатическую силу для создания крутящего момента, который циклически повторяется вследствие смещения центра тяжести циклического агрегата. Тела (116) поплавка подвергаются действию выталкивающей силы в жидкости (134), которая вызывает движение (146) тел поплавка вверх и переводит их в верхнее положение. Движение вверх приводит в действие приводной элемент. Движение вверх, кроме того, заставляет воздух вытесняться из верхних сегментов (102а, 102d) деформируемого элемента (100) в нижние сегменты (102b, 102с), и в результате центр тяжести циклических блоков, составляющих тела поплавка, (116), а деформируемый элемент (100) поднят над осью вращения (106). Когда тела поплавка (116) перешли в свое верхнее положение, указанное приподнятое положение центра тяжести позволяет повернуть циклические блоки в их начальное положение.
Тип:
ЗаявкаПодано:
5 марта 2012 г.Дата публикации:
20 марта 2014 г.Изобретатель:
Маджид Рахманифар
ПНЕВМАТИЧЕСКИЙ ЭЛЕКТРОГЕНЕРАТОР
Номер публикации: 20140049047
Резюме: Пневматический электрический генератор включает в себя воздушный компрессор, электродвигатель для питания воздушного компрессора и резервуар для хранения сжатого воздуха. Вентилятор, клапан и труба направляют выпущенный воздух на вентилятор, тем самым вращая вентилятор и два электрических генератора. Аккумулятор и зарядное устройство обеспечивают питание электродвигателя, так что электрический генератор с пневматическим приводом можно переместить в заранее выбранное место для обеспечения электроэнергией способом, свободным от угарного газа и загрязнения.
Тип:
ЗаявкаПодано:
15 августа 2012 г.Дата публикации:
20 февраля 2014 г.Изобретатель:
Сами Мохаммед Аленези
МОБИЛЬНОЕ ГИДРОЭЛЕКТРИЧЕСКОЕ УСТРОЙСТВО
Номер публикации: 20130307272
Реферат: Мобильное гидроэлектрогенераторное устройство. Гидроэлектростанция может включать резервуар для воды с подвешенным внутри него турбогенератором. Вода может перекачиваться со дна резервуара для воды и в верхнюю часть резервуара для воды. Вращающаяся подача воды может быть направлена на лопасти ротора турбогенератора и тем самым вырабатывать электроэнергию.
Тип:
ЗаявкаПодано:
7 мая 2013 г.Дата публикации:
21 ноября 2013 г.Изобретатель:
Морис Леон Смит
Система использования центробежной силы и способ выработки электроэнергии
Номер публикации: 20130247551
Реферат: Раскрытое изобретение представляет собой описание средств для создания условий, при которых жидкая материя (показанная вода) подвергается воздействию центробежной силы, вызванной вращением воды при ее циркуляции в замкнутая система со скоростями, достаточными для выработки электроэнергии. Энергия, генерируемая системой, используется для поддержания вращения всей системы, непрерывной циркуляции воды внутри вращающейся системы и подачи избыточной электроэнергии для произвольного использования.
Тип:
ЗаявкаПодано:
26 марта 2012 г.Дата публикации:
26 сентября 2013 г.Изобретатель:
Джон Чарльз Хейсс
ЭНЕРГЕТИЧЕСКИЙ БЛОК
Номер публикации: 20130221673
Реферат: Энергоустановка на основе самогенерирующих источников энергии, реализованная комбинацией бассейна, водяного колеса, водонаправляющего канала, двигателя, гидроизоляционного корпуса двигателя, водокомпрессионной трубы. , водяной винт и электрогенератор, а также электрический блок управления; с помощью этого энергоблока небольшая часть собственной энергии может быть использована для выработки электроэнергии без необходимости использования дополнительных источников топлива или энергии; таким образом, это всепогодный, наиболее экономичный и экологически безопасный энергоблок с более длительным сроком службы и меньшей занимаемой площадью, а также с минимальными затратами на защиту, техническое обслуживание и управление.
Тип:
ЗаявкаПодано:
23 февраля 2012 г.Дата публикации:
29 августа 2013 г.Изобретатель:
Пинг-Чи Лю
МНОЖИТЕЛЬ КИНЕТИЧЕСКОЙ ЭНЕРГИИ РОТАЦИОННЫЙ ДВИГАТЕЛЬ
Номер публикации: 20130202438
Резюме: Роторный двигатель с множителем кинетической энергии, который работает за счет увеличения умноженной силы кинетической энергии, которая начинается благодаря своему ускорителю и вращательному движению его масс разного размера, что приводит к вращению кинетической энергии. двигатель имеет в своей конструкции. Целью настоящего изобретения является обеспечение энергией, с тем чтобы движущееся и/или электрическое энергетическое оборудование могло обеспечивать более практичную, чистую, менее дорогостоящую и менее загрязняющую энергию, чем существующие сегодня виды энергии, поскольку последние являются дорогостоящими и сильно загрязняющими окружающую среду. . Эта цель достигается благодаря простой инновационной конструкции, основная характеристика которой состоит из масс и магнитов, обеспечивающих вращательное движение на валу двигателя.
Тип:
ЗаявкаПодано:
16 апреля 2012 г.Дата публикации:
8 августа 2013 г.Изобретатель:
Хесус Каррильо Гальван
Глубоководная гидроэлектростанция
Номер публикации: 20130187386
Реферат: Глубоководная энергетическая система включает в себя первоначально эвакуированную сферу, имеющую стенки достаточной прочности или усиления для сохранения ее структурной целостности при глубоководных давлениях; силовая ось, проходящая от ее северного полюса до южного полюса сферы; лопатки турбины закреплены на опорной раме, закрепленной на оси в широтной плоскости сферы; и впускные отверстия, расположенные в плоскости лопастей турбины и принимающие приток окружающей глубокой воды против лопасти. Поток жидкости приводит во вращение силовую ось лопастей, закрепленную на раме. Упорная платформа жестко закреплена внутри сферы на силовом мосту и над турбиной, а генератор закреплен на опорной платформе на южном полюсе сферы, спирально вытесняющей воду из южного полюса системы.
Тип:
ЗаявкаПодано:
24 января 2013 г.Дата публикации:
25 июля 2013 г.Изобретатели:
Лакдас Нанаяккара, Правин Нанаяккара
СИСТЕМА ДЛЯ ПРОИЗВОДСТВА ЭЛЕКТРИЧЕСКОЙ И МЕХАНИЧЕСКОЙ ЭНЕРГИИ С ИСПОЛЬЗОВАНИЕМ ТЕПЛОВОГО ГРАДИЕНТА
Номер публикации: 20130140830
Реферат: Система для выработки электрической и механической энергии с использованием градиента температуры включает устройство для создания градиента температуры, сообщающееся с турбогенератором. Система предназначена для поглощения тепловой энергии из внешней среды, ее циркуляции через циркуляционную камеру и преобразования тепловой энергии в одну или обе из механической силы и электричества, которые могут быть возвращены в саму систему или использованы другими устройствами.
Тип:
ЗаявкаПодано:
6 декабря 2011 г.Дата публикации:
6 июня 2013 г.Изобретатель:
ДОНАЛЬД Э. ХИНКС
Гидравлический генератор энергии и гидравлическая система производства энергии
Номер публикации: 20130043684
Реферат: Гидравлический генератор простой конструкции, легко адаптируемый к различным источникам воды и различным электрическим генерирующим мощностям, включая поплавок, плавающий на поверхности воды в резервуаре с водой, и механизм преобразования линейного вращения, который преобразует линейное движение во вращательное движение, и гидравлическая генерирующая система, использующая это устройство.
Тип:
ЗаявкаПодано:
9 августа 2012 г.Дата публикации:
21 февраля 2013 г.Изобретатель:
Тору Шинохара
СИСТЕМА ПРОИЗВОДСТВА ГИДРАВЛИЧЕСКОЙ ПЕРЕХОДНОЙ ЭНЕРГИИ
Номер публикации: 20130038062
Реферат: Раскрыта система для получения гидравлической переходной энергии, основанная на принципе гидравлических переходных процессов, включающих преобразование кинетической энергии в потенциальную энергию (давления), которая будет служить надежным, возобновляемым , недорогой и экологически чистый источник энергии, а также обеспечить хорошие экологические преимущества (и кредит CO2) за счет существенного сведения к минимуму выбросов парниковых газов. Для использования потенциальной энергии (давления), выработанной в системе, изобретение обеспечивает непрерывный и устойчивый переходный скачок давления. Быстродействующие клапаны с соответствующими и совместимыми системами контрольно-измерительных приборов позволяют периодически и непрерывно создавать скачки давления для поддержания высокого давления на выходе из системы. Стабильное повышение давления на выходе из системы можно использовать для привода турбины для выработки электроэнергии или для перекачивания жидкости от более низкого давления к более высокому давлению, при этом его можно использовать для привода насосов, компрессоров и т.п., для которых требуется энергия. ввод для их работы.
Тип:
ЗаявкаПодано:
27 июля 2012 г.Дата публикации:
14 февраля 2013 г.Изобретатель:
Самусидин Адевале Салу
ЭЛЕКТРОГЕНЕРАТОРНАЯ СИСТЕМА ДЛЯ Улавливания энергии ветра на движущемся транспортном средстве
Номер публикации: 20130038066
Abstract: Раскрыта система для системы электрического генератора для транспортного средства. Система включает ветряную турбину, электрический генератор, механически соединенный с ветряной турбиной и сконфигурированный для подключения к накопителю электрической энергии, который сконфигурирован для хранения электрической энергии на борту транспортного средства, и жесткий конический корпус, образующий внутреннюю камеру. , корпус имеет впускной конец и выпускной конец, причем впускной конец имеет больший диаметр, чем выпускной конец, и конический корпус выполнен с возможностью направления потока ветра в ветровую турбину.
Тип:
ЗаявкаПодано:
10 августа 2011 г.Дата публикации:
14 февраля 2013 г.Изобретатель:
ХОСЕ ДИАС
УСТРОЙСТВО ГЕНЕРАЦИИ КИНЕТИЧЕСКОЙ ЭНЕРГИИ
Номер публикации: 20130001949
Реферат: Устройство для выработки кинетической энергии включает в себя механизм вращения, ускоритель скорости, соединенный с механизмом вращения, и элемент генерирования мощности, соединенный с ускорителем. Механизм вращения включает в себя центральный вал и внешнюю окружную зону, окружающую центральный вал. Внешняя периферийная зона принимает в себя перегородки, установленные в радиальном направлении и отстоящие друг от друга так, что перегородки имеют наклон вверх. Центральный вал соединен с возможностью вращения с устройством увеличения скорости, а устройство увеличения скорости соединено с элементом генерирования энергии, образуя устройство генерирования кинетической энергии. Механизм вращения расположен под потоком воды, чтобы позволить каждой из приемных зон принимать вес воды, так что вес воды заставляет механизм вращения вращаться в заданном направлении и, таким образом, приводит в движение преобразователь скорости для выработки электрического тока. сила.
Тип:
ЗаявкаПодано:
17 февраля 2011 г.Дата публикации:
3 января 2013 г.Изобретатель:
Сянь-Мин Линь
ГИДРАВЛИЧЕСКИЙ ГЕНЕРАТОР
Номер публикации: 20120312008
Реферат: Изобретение относится к гидрогенератору, образованному набором различных специфических цепей, которые координируются и дополняют друг друга, например, позволяют поплавкам входить в бак и принуждать указанные поплавки двигаться заданный путь, чтобы максимально использовать его восходящую силу. Первая цепь нейтрализует внутреннее гидравлическое давление на переднюю часть поплавков, позволяя установить поплавки на дно резервуара. Как только поплавок попадает в резервуар, первая цепь передает поплавок второй цепи, которая использует восходящую силу поплавков и преобразует эту силу в механическую работу в соответствии с принципом Архимеда. Вторая цепь приводит в движение всю систему. Третья цепь удаляет каждый поплавок, достигший поверхности из резервуара, и вводит указанный поплавок во внешний канал, который возвращает поплавок к входу, с помощью четвертой цепи и системы среднего давления, которая возвращает поплавок обратно. повторяется автоматически и бесконечно.
Тип:
ЗаявкаПодано:
13 января 2011 г.Дата публикации:
13 декабря 2012 г.Изобретатель:
Серхио Кампос Альварес
Аппарат для получения электрической энергии
Номер публикации: 20120299304
Реферат: Устройство для выработки электроэнергии включает в себя базовый уровень и приподнятую платформу с расположенным на приподнятой платформе резервуаром для воды. Источник воды расположен на уровне основания, в то время как множество относительно небольших насосов и трубопроводов соединяют источник воды с каждым из насосов и каждый из насосов с резервуаром для воды на платформе. Источник электрической энергии предусмотрен для питания насосов для перекачивания воды вверх из источника воды в резервуар для воды. Кроме того, установка включает в себя множество конструктивных опор, расположенных по меньшей мере на трех различных уровнях, промежуточных на уровне основания, и приподнятую платформу с водяным колесом, включающую в себя множество радиально расположенных и проходящих в поперечном направлении пластинчатых вентиляторов, расположенных один под другим на структурные опоры. Наконец, к каждому из водяных колес оперативно подключен электрический генератор.
Тип:
ЗаявкаПодано:
26 мая 2011 г.Дата публикации:
29 ноября 2012 г.Изобретатель:
Абдулрейда Аль-Саффар
Бесщеточный двигатель постоянного тока с однонаправленным питанием, включая выходную обмотку переменного напряжения и двигательную систему
Номер патента: 8294317
Реферат: Однонаправленный бесщеточный двигатель постоянного тока включает диск, прикрепленный к раме, множество пластинчатых постоянных магнитов, расположенных на диске через равные промежутки вокруг диска, магнитные сердечники закреплены к каркасу в соответствии с множеством постоянных магнитов, обмотки, каждая из которых намотана вокруг магнитопровода и на которые подается питание постоянного тока, заданного количества магнитопроводов, закрепленных на каркасе, и обмоток, каждая из которых намотана вокруг магнитопровода ядра и подключен к устройству потребления энергии. Постоянные магниты расположены так, что угол, образованный прямой линией, проходящей через центр диска и центр постоянного магнита, и нормалью, проходящей через центр плоскости магнитного полюса постоянного магнита, составляет от 0° до 60°. °.
Тип:
ГрантПодано:
9 ноября 2007 г.Дата выдачи патента:
23 октября 2012 г.Изобретатели:
Ёсихидэ Камида, Ясухиро Комацу
Автоматический генератор энергии
Номер публикации: 20120235527
Резюме: Один из вариантов автоматизированного генератора энергии имеет одну базовую плату (10), три магнитные платы (48), две платы катушек (58), приводные платы (68), двигатель постоянного тока. (70) с комплектом передаточного устройства, состоящего из двух шкивов (70А), (70В) и ремня (70С), и тормоза (80). Базовая доска (10), катушки (58), неподвижная доска (64) приводных досок (68) установлены на четырех резьбовых стержнях (20), а магнитные доски (48) и поворотная доска (62) приводной доски (68). ) установлены на валу (30), основная плата (10) находится в самом низу, а приводные платы (68) — вверху, всего между ними пять слоев — две катушки (58), зажатые тремя магнитными досками (48). ). Описаны и показаны другие варианты осуществления.
Тип:
ЗаявкаПодано:
17 марта 2011 г.Дата публикации:
20 сентября 2012 г.Изобретатель:
Тай Коан Ли
Гидроэлектрический генератор Sipoline
Номер публикации: 20120200088
Реферат: Переносной гидроэлектрогенератор с собственным водоснабжением. Трубка с первым отверстием на одном конце для приема жидкости из желоба и вторым отверстием на другом конце для выпуска полученной жидкости. Трубка имеет форму перевернутой буквы U, сторона А которой проходит от первого отверстия к вершине перевернутой буквы U, а сторона В, которая включает в себя турбину, соединенную с генератором, проходит от вершины перевернутой буквы U ко второму концу. Сторона А имеет объем, который меньше объема стороны В, а второе отверстие расположено выше уровня жидкости в желобе. Когда жидкость находится в трубке, ее вес на стороне B больше, чем ее вес на стороне A, что заставляет жидкость течь из первого отверстия через трубку и из второго отверстия.
Тип:
ЗаявкаПодано:
3 февраля 2011 г.Дата публикации:
9 августа 2012 г.Изобретатель:
Акино Андриан Харрис
ГЕОМАГНИТНЫЙ ГЕНЕРАТОР С ПОМОЩЬЮ ГРАВИТАЦИИ
Номер публикации: 20120098272
Abstract: Устройство 100 для выработки геомагнитной энергии имеет направляющее средство 20, один или несколько подвижных постоянных магнитов 10, множество катушек 40 и батарею 50 или серию батарей. Один или более подвижных постоянных магнитов 10, каждый постоянный магнит 10 имеет северную полярность N на первом конце 11 и южную полярность S на противоположном втором конце 12. Один или более магнитов 10 расположены и направляются по направляющей траектории GP. посредством направляющего средства 20. Каждая катушка 40 из множества катушек 40 расположена вокруг направляющего средства 20, окружая как направляющее средство 20, так и направляющую траекторию GP, по которой перемещаются один или несколько постоянных магнитов 10. Каждая катушка 40 имеет поперечное сечение с увеличивающимся числом витков, проходящих от первого конца 44 с минимальной обмоткой до второго конца 45 с максимальной обмоткой. Батарея 50 или ряд батарей соединены с множеством катушек 40.
Тип:
ЗаявкаПодано:
25 октября 2010 г.Дата публикации:
26 апреля 2012 г.Изобретатель:
Путалат Корот Рагхупрасад
Генератор гравитационной плавучести
Номер публикации: 20120085088
Реферат: Устройство и способ получения вечной энергии от гравитации или любой другой энергии с тем же эффектом с помощью вращающейся цилиндрической системы, состоящей из трех подсистем (грузовой, средней и жидкостной) и в зависимости от влияние гравитации на вес и плотность используемых материалов. Система в основном направлена на преобразование энергии гравитации в энергию плавучести и создание эффекта энергии плавучести больше, чем влияние энергии гравитации на баланс всей системы.
Тип:
ЗаявкаПодано:
7 октября 2010 г.Дата публикации:
12 апреля 2012 г.Изобретатель:
Валид Абульхаггаг Ибрагим Ахмед Зидан
Электрический генератор, использующий распыление жидкости под высоким давлением для вращения выработки электроэнергии
Номер публикации: 20120080882
Abstract: Изобретатель представляет новый генератор, в котором используется новый метод вращения механизмов выработки электроэнергии, содержащий генератор переменного тока с постоянными магнитами предшествующего уровня техники, прикрепленный к цилиндрическому корпусу, центральный вал ротора которого проникает внутрь вниз через центральное отверстие через верхнюю часть бака внутри бака. Роторное колесо соединено с валом ротора, чтобы способствовать вращению ротора. Новый способ вращения указанного роторного колеса дополнительно включает насос для жидкости, посредством которого жидкость вытягивается из нижней части резервуара через выходное отверстие резервуара и возвращается через входное отверстие резервуара в верхней части резервуара, при этом наконечник для жидкости под высоким давлением имеет небольшое отверстие в центре. дополнительно соединен внутри бака. Когда отработанная жидкость вытягивается со дна резервуара через насос, она затем проталкивается через наконечник высокого давления, в результате чего высокоскоростные брызги отталкиваются от роторного колеса, реагируя с лопастями роторного колеса, заставляя ротор вращаться, создавая полезную электроэнергию.
Тип:
ЗаявкаПодано:
4 октября 2010 г.Дата публикации:
5 апреля 2012 г.Изобретатель:
Кеннет Тайрон Ньюберн
БЕГ ОКЕАНСКОЙ ВОДЫ И ВЕРТИКАЛЬНЫЙ ПАДЕНИЕ ЭТО ЭНЕРГЕТИЧЕСКАЯ ГЕНЕРАЦИЯ
Номер публикации: 20120061967
Реферат: Это изобретение разработано с использованием всех законов гидравлики и гидродинамики путем рытья туннеля, прокладки трубы и т. д. в земле, окружающей водоем, на различных уровнях, по крайней мере, начиная с поверхность водоема и переход к самому дну водного ложа в различных геометрических фигурах, углах и градусах, подходящих для индивидуального проекта и индивидуального участка и индивидуальных требований генерации в любой точке мира, чтобы вода вышла из воды тела либо со дна, ниже дна водоема и выступающие в водоем, либо с боков, окружающих водоем, и впадают в сооружение из входа и различных входов, устроенных ниже основного уровня водоема, или средний уровень моря или основной уровень водоема на земле и необходимый в водоеме также для того, чтобы различные турбогенераторы различной мощности, формы, конструкции, доступные на мировых рынках, возводятся каскадным способом или любой другой системой, предназначенной для
Тип:
ЗаявкаПодано:
22 апреля 2010 г.Дата публикации:
15 марта 2012 г.Изобретатели:
Шринивас Бхаскар Чаганти, К. Бала
Поршневой двигатель с магнитным приводом и процесс с использованием обратного магнитного переключения
Номер публикации: 20120007447
Реферат: Поршневой двигатель с магнитным приводом использует накопленную энергию магнитов, особенно редкоземельных магнитов, и электромагнитное поле для возвратно-поступательного движения магнитного привода. Преобразовательный механизм, такой как шатун и коленчатый вал, преобразует возвратно-поступательное движение магнитного привода во вращательное движение для приведения в действие рабочего объекта. Соленоид, состоящий из неферромагнитного золотника, имеющего трубчатую центральную часть с катушкой проволоки, намотанной вокруг центральной части, соединен с источником питания и переключающим механизмом. Механизм переключения переключает магнитную полярность на концах соленоида, чтобы попеременно отталкивать и притягивать постоянные магниты на концах магнитного привода. Вал, соединяющий магниты, проходит через центральную часть соленоида. Механизм управления, соединяющий выходной вал и механизм переключения, обеспечивает синхронизацию для переключения полярности соленоида для приведения в действие магнитного исполнительного механизма.
Тип:
ЗаявкаПодано:
8 июля 2010 г.Дата публикации:
12 января 2012 г.Изобретатель:
Кендалл С. Госвенер
Система колес с гидравлическим приводом для производства электроэнергии
Номер публикации: 20120001433
Реферат: Колёсная система с гидравлическим приводом для выработки электроэнергии состоит из двойного колёсного узла, имеющего пару идентичных колёс, установленных вертикально на горизонтальной оси. Сосуды для жидкости соединены с колесами, прилегающими к периметрам колес, и соединяют колеса между собой. Каждый приемник для жидкости имеет одно отверстие для приема жидкости, когда приемник для жидкости находится в первой ориентации, и для опорожнения жидкости из приемника для жидкости, когда приемник для жидкости находится во второй ориентации. Система колес с гидравлическим приводом дополнительно содержит пару инерционных колес. Каждое инерционное колесо расположено на соответствующей стороне колесного узла и установлено на горизонтальной оси так, что колесный узел вращается вместе с инерционными колесами. Каждое инерционное колесо имеет множество равноотстоящих друг от друга утяжеляющих элементов, которые обеспечивают плавное непрерывное движение узла колеса.
Тип:
ЗаявкаПодано:
4 июля 2010 г.Дата публикации:
5 января 2012 г.Изобретатель:
Антуан Мовад
ВЕТРОЗАРЯДНОЕ УСТРОЙСТВО ДЛЯ АВТОМОБИЛЬНЫХ АККУМУЛЯТОРОВ
Номер публикации: 20110316470
Реферат: В одном иллюстративном варианте ветряная система зарядки включает ветряное вращательное устройство, соединенное с вращающимся валом, и множество электрических генераторов, расположенных в различных продольных точках вдоль вращающийся вал и каждый из множества электрических генераторов приводятся во вращение одновременно вращающимся валом. Благодаря тому, что электрические генераторы расположены в разных местах в продольном направлении, большее количество электрических генераторов может одновременно приводиться в действие общим валом. В некоторых случаях контроллер может быть сконфигурирован так, чтобы позволить большему количеству электрических генераторов обеспечивать электрический ток для перезарядки батареи, когда скорость вращения вращающегося вала увеличивается, и может отключать большее количество электрических генераторов, чтобы не обеспечивать электрический ток. когда скорость вращения вращающегося вала уменьшается.
Тип:
ЗаявкаПодано:
29 июня 2010 г.Дата публикации:
29 декабря 2011 г.Изобретатель:
Ричард Роджерс
УСТРОЙСТВО ДЛЯ ПРОИЗВОДСТВА ЭЛЕКТРОЭНЕРГИИ С НЕСКОЛЬКИМИ ПОСЛЕДОВАТЕЛЬНЫМИ ТЕПЛОВЫМИ НАСОСАМИ
Номер публикации: 20110309635
Abstract: Устройство для выработки электроэнергии (1) содержит: первый тепловой насос (3), снабженный первым замкнутым контуром (15), в котором циркулирует первый жидкий теплоноситель, и с первым теплообменником (17) между первым теплоносителем и потоком атмосферного воздуха, в котором поток атмосферного воздуха передает некоторое количество тепла первому теплоносителю, по меньшей мере, вторым тепловым насосом (5) , снабженный вторым замкнутым контуром (23), в котором циркулирует второй жидкий теплоноситель, и вторым теплообменником (25) между вторым жидким теплоносителем и третьим жидким теплоносителем, в котором циркулирует второй жидкий теплоноситель. текучая среда передает некоторое количество тепла третьему жидкому теплоносителю; средство для передачи количества тепла от первого жидкого теплоносителя ко второму жидкому теплоносителю; третий замкнутый контур (9), в котором циркулирует третий теплоноситель; турбину (11), вставленную в третий замкнутый контур (9) и приводимую в действие третьим жидким теплоносителем; электрический генератор
Тип:
ЗаявкаПодано:
18 декабря 2009 г.Дата публикации:
22 декабря 2011 г.Заявитель:
XEDA INTERNATIOANALИзобретатель:
Альберто Сардо
ЗЕЛЕНЫЙ ЭЛЕКТРОМОБИЛЬ, ИСПОЛЬЗУЮЩИЙ НЕСКОЛЬКО ИСТОЧНИКОВ ЭНЕРГИИ
Номер публикации: 20110309786
Abstract: Автономное экологичное транспортное средство без двигателя с электроприводом, содержащее по меньшей мере один электродвигатель, соединенный с ведущей осью, и множество аккумуляторных батарей для обеспечения накопленной электрической энергии указанного мотор. Упомянутый двигатель приводится в движение, и указанные батареи заряжаются от нескольких источников энергии. Транспортное средство может включать в себя систему сбора и ускорения ветра, используемую для приведения в действие по меньшей мере одной ветряной турбины или аналогичной для выработки электроэнергии. Сюда также могут входить солнечные и тепловые рецепторы, средства для выработки электроэнергии из солнечных и тепловых источников. Кроме того, транспортное средство может включать по меньшей мере один электрический генератор, соединенный с другой осью, при этом вращение оси с передаточным механизмом увеличивает скорость вращения генератора для выработки электроэнергии.
Тип:
ЗаявкаПодано:
18 июня 2010 г.Дата публикации:
22 декабря 2011 г.Изобретатель:
М. Хассан Хассан
Генерирующая система водного цикла
Номер публикации: 20110304147
Реферат: Генераторная система включает в себя основание, первый резервуар для хранения воды, установленный на нижней части основания, водонапорную башню, установленную на верхней части основания, как минимум один генератор блок, установленный на основании и расположенный между водонапорной башней и первым резервуаром для воды, напорный блок, соединенный между водонапорной башней и первым резервуаром для воды и проходящий через генераторную установку, и по меньшей мере одно водоподъемное устройство, соединенное между первым резервуар для хранения воды и водонапорная башня для подъема воды из первого резервуара для хранения воды в водонапорную башню. Таким образом, водоподъемное устройство может перекачивать воду из первого резервуара для хранения воды в водонапорную башню, не потребляя электроэнергии, что значительно снижает затраты.
Тип:
ЗаявкаПодано:
14 июня 2010 г.Дата публикации:
15 декабря 2011 г.Изобретатель:
Фу Хунг Хо
Способ и механизм сетевого устройства сервера связи ветряной турбины на солнечной панели
Номер публикации: 20110270682
Резюме: Метод, включенный в настоящее описание, представляет собой механизм узлового устройства сервера связи ветровой турбины с беспроводной солнечной панелью на альтернативной энергии со встроенным массивом серверов связи, беспроводной энергией и механизмом связи с защитой, совместным использованием, хранением, доступ, аутентификация, управление батареей, сертификация, обработка прикрепления и механизмы отслеживания. Метод и механизм используются через сетевые серверы, солнечные панели и беспроводные электронные устройства (онлайн и оффлайн), а также мобильные (беспроводные) устройства связи, встроенные в один водонепроницаемый, температурный, контролируемый аппаратный пакет с самоопределением. Способ и механизм также работают как часть банковской системы киловатт-часов и механизма теплового пассивного солнечного контроля для жилых помещений.
Тип:
ЗаявкаПодано:
28 апреля 2010 г.Дата публикации:
3 ноября 2011 г.Изобретатель:
Дэвид Валин
ДРЕВНЯЯ ГИДРОЭЛЕКТРИЧЕСКАЯ КОМПАНИЯ
Номер публикации: 20110266804
Резюме: Древняя гидроэлектростанция включает портативную гидроэлектростанцию. Потребителям предоставляется экономически эффективная и надежная система обеспечения практически чистой выработки электроэнергии без перекрытия источника воды и негативного воздействия на окружающую среду. Высокоэффективная портативная гидроэлектростанция предназначена для использования с движущимся источником воды. В системе используется антифриз с водой, чтобы обеспечить более термически эффективную рабочую жидкость, используемую для повышения давления и объемной емкости системы при использовании.
Тип:
ЗаявкаПодано:
13 апреля 2011 г.Дата публикации:
3 ноября 2011 г.Изобретатель:
Джозеф Дольчимасколо
Вертикальный резервуар для жидкости с бесконечной цепью
Номер патента: 8042334
Реферат: Настоящее изобретение относится к устройству и связанным с ним способам генерирования энергии путем улавливания и использования энергии, генерируемой любым количеством воздуха, всплывающего в воде. Устройство включает в себя рамную конструкцию, на которой с возможностью вращения установлены верхнее ведущее колесо, нижнее колесо и вертикальный контейнер для столба жидкости. На верхние и нижние колеса насажена бесконечная цепь элементов газовой капсулы. Эта бесконечная цепь проходит вертикально вверх в контейнер столба текучей среды и проходит через него через уплотнительное отверстие в дне контейнера. Поскольку бесконечная цепь элементов газовой капсулы проходит вертикально через жидкость в контейнере, давление жидкости на элементы из-за высоты столба жидкости в контейнере создает чистую выталкивающую силу вверх на элементах, заставляя их подниматься, создавая кинетическая энергия, вращающая колеса.
Тип:
ГрантПодано:
26 сентября 2008 г.Дата выдачи патента:
25 октября 2011 г.Изобретатель:
Ренато Бастос Рибейро
Автономный приводной магнитный генератор переменного тока и генератор с прямым приводом на маховике, также известный как MAW-генератор с прямым приводом
Номер публикации: 20110254398
Резюме: MAW-DirectDrives Генераторы с маховиками представляют собой автономные крупногабаритные маховики с прямым приводом, подвешенные внутри стационарного корпуса, прикрепленного под прочной ведущей пластиной и шпиндельным узлом, соединенным с корпусом посредством центральная ступица колеса в сборе. Вращающийся маховик приводится в движение одним из четырех-десяти бескаркасных статоров генератора переменного тока с прямым приводом, прикрепленных к верхней части корпуса настроенными и согласованными парами, причем каждая пара отделена друг от друга с двухсторонним приводом-ротором с постоянными магнитами, соединенными с приводом. -пластина. Маховик используется для распределения рабочей нагрузки на большую площадь, чтобы получить возможность задействовать в работе больше статоров и преобразовать кинетическую энергию, присутствующую во вращающемся маховике, за счет работы дополнительных статоров, работающих более эффективно в их настроенной и согласованной конфигурации. Электрический вход, необходимый для поддержания скорости маховика, регенерируется в процессе работы, повышая плотность выходного тока.
Тип:
ЗаявкаПодано:
20 апреля 2010 г.Дата публикации:
20 октября 2011 г.Изобретатель:
Дана Аллен Хансен
Ветроэнергетическая система непрерывного действия со вспомогательными лопастями
Номер публикации: 20110248512
Реферат: Один из вариантов осуществления ветроэнергетической системы непрерывного действия с вспомогательными лопастями состоит из основного генератора (150) и шести комплектов вспомогательных лопастей (120), каждый из которых приводится в движение двигателем (130). своего собственного. Электродвигатели (130) питаются от батарей, солнечных батарей или части мощности, вырабатываемой генератором (150). Когда двигатели (130) включены, наборы вспомогательных лопастей (120) перемещаются и создают силу, перпендикулярную выдвинутому рычагу (110В), на котором он установлен, и возникающий крутящий момент способствует вращательному движению всего монтажного устройства (110). Передаточное устройство передает вращательное движение монтажного устройства (110) комплектов (120) вспомогательных ножей и двигателей (130) на вал генератора (150А) и приводит к выработке электроэнергии.
Тип:
ЗаявкаПодано:
7 апреля 2010 г.Дата публикации:
13 октября 2011 г.Изобретатель:
Тай Коан Ли
РАСШИРЯЕМЫЙ МНОГОКОМПЛЕКТНЫЙ ЦИРКУЛЯЦИОННЫЙ СПОСОБ И СИСТЕМА ПРОИЗВОДСТВА ЭНЕРГЕТИЧЕСКОЙ ЭНЕРГЕТИКИ
Номер публикации: 20110241345
Реферат: Расширяемый многоагрегатный циркуляционный способ и система производства гидроэлектроэнергии. Система включает зону управления электростанцией (А), зону трансформаторной подстанции (В), циркуляционную цистерну (С), снабженную множеством малых цистерн (от СА1 до СА20). В малых цистернах (СА1-СА20) установлены ворота (Е1-Е20) и защитные экраны. Зона станции водяных насосов снабжена множеством водяных насосов (от D1 до D20). Водяные насосы (от D1 до D20) подключены к малым цистернам (от CA1 до CA20). Малые цистерны (от СА1 до СА20) соединены с напорными трубами (от G1 до G20). Напорные трубы (от G1 до G20) подсоединяются к рабочим трубам (от h2 до h20). Несколько генераторных установок с водяными турбинами (от F1 до F51) и множество затворов (E42, E43, E47, E50, E53, E56, E59)., Е63, Е64, Е65) устанавливаются в рабочие трубы (от h2 до h20).
Тип:
ЗаявкаПодано:
9 сентября 2008 г.Дата публикации:
6 октября 2011 г.Изобретатель:
Вэйпин Чен
perpetual self starting magnet motor
TikTok
Upload
For You
Following
adrtech50
Cezar Florica
#fyp #satisfying #viral #fypシ #learnontiktok #foryoupage
81,1 тыс. лайков, 3,5 тыс. комментариев. Видео TikTok от Cezar Florica (@adrtech50): «#fyp #satisfying #viral #fypシ #learnontiktok #foryoupage». Самозапускающийся магнитный двигатель 100% | Настоящий вечный двигатель. Бегущий по лезвию 2049.
2,4 млн просмотров|
Blade Runner 2049 — Synthwave Goose
timothykomadowski
Timk
#perpetualmotion #freeenergy #magnetmotor #gasprices #fuel
113 Likes, 20 Comments. Видео TikTok от Timk (@timothykomadowski): «#perpetualmotion #freeenergy #magnetmotor #gasprices #fuel». Приводной магнит | У шестерни недостаточно крутящего момента, чтобы установить магнит на место. | Поэтому мне пришлось опустить его примерно на 30-45 градусов | …. Пропустите голландца.
4991 просмотр|
Pass the Dutchie — Jeremy Oates & The Music Makers
adrtech50
Cezar Florica
#foryoupage #knowledge #learnontiktok #viral #fypシ #satisfying #fyp
29,3 тыс. лайков, 1,1 тыс. комментариев. Видео TikTok от Cezar Florica (@adrtech50): «#foryoupage #knowledge #learnontiktok #viral #fypシ #satisfying #fyp». Самозапускающийся магнитный двигатель 100%🔥. Бегущий по лезвию 2049.
939 тыс. просмотров|
Blade Runner 2049 — Synthwave Goose
timothykomadowski
Timk
#PrimeDayDreamDeals #perpetualmotion #freeenergy #magnetmotor #gasprices #fuel #belairdirectdrivechallenge
28. 4K Likes, 830 комментариев. Видео TikTok от Timk (@timothykomadowski): «#PrimeDayDreamDeals #perpetualmotion #freeenergy #magnetmotor #pricesgas #fuel #belairdirectdrivechallenge». В этом видео поворотный рычаг самостоятельно перемещается на 315°. | На самом деле я не толкаю руку, а двигаю ее, пока она не взлетит сама по себе | Еще 45° и эта присоска вечная | …. Кумбия Буэна.
869,8 тыс. просмотров|
Cumbia Buena — Grupo La Cumbia
эндридушку487
научный эксперимент сумасшедший
Магнитный вечный двигатель? Magnetoelectric #FYP #AMOUNT Energy #MOTOR #HANDMADE #DIYPROJECT #TREND #FreeEenge #FRAFT #FREEENERGE #RAFT #FREEENERGE № #FREEENERGE . Видео TikTok из научного эксперимента madman (@endridushku487): «Магнитный вечный двигатель? Грязные мысли.
13,8 тыс. просмотров|
Dirty Thoughts — Chloe Adams
endridushku487
science experiment madman
Small magnet engine #fpy #perpetualmotion #perpetua #Rotating #power #fpyシ
479 Likes, 6 комментариев. Видео TikTok из научного эксперимента Madman (@endridushku487): «Маленький магнитный двигатель #fpy #perpetualmotion #perpetua #Rotating #power #fpyシ». Танец в темноте (Дип Хаус).
18,2 тыс. просмотров|
Dance In The Dark (Deep House) — 阿增&苏七岁&气质隆Sir
timothykomadowski
Timk
#perpetualmotion #freeenergy #magnetmotor #gasprices #fuel
411 лайков, 62 комментария. Видео TikTok от Timk (@timothykomadowski): «#perpetualmotion #freeenergy #magnetmotor #gasprices #fuel». Теперь на нем 4 приводных магнита, и вы можете видеть, | Просто не хватает движения против часовой стрелки | Я попробовал магнит 5-го привода, просто удерживая его на месте | …. Высшее состояние сознания (Tweekin Acid Funk).
69 тыс. просмотров|
Higher State of Consciousness (Tweekin Acid Funk) — Josh Wink
timothykomadowski
Timk
#perpetualmotion #magnet #magnetmotor #fuel #gasprices #freeenergy #freeenergygenerator #adamsavage #bojacksonsmyhero
122 лайков, 33 комментария. Видео TikTok от Timk (@timothykomadowski): «#perpetualmotion #magnet #magnetmotor #fuel #gasprices #freeenergy #freeenergygenerator #adamsavage #bojacksonsmyhero». 6 приводных магнитов | Пожалуйста, следуйте!! | 1 легкое прикосновение пальца | …. В Большой Стране.
2769 просмотров|
In A Big Country — Big Country
timothykomadowski
Timk
#perpetualmotion #freeenergy #magnetmotor #gasprices #fuel #PrimeDayDreamDeals
10.7K Likes, 285 Comments. Видео TikTok от Timk (@timothykomadowski): «#perpetualmotion #freeenergy #magnetmotor #pricesgas #fuel #PrimeDayDreamDeals». 1 жесткий толчок, чтобы увидеть, что произойдет | Далее я делаю 8 рук. | 7 будет толкать против часовой стрелки | …. Перестрелка.
483,7 тыс. просмотров|
Shootout — Izzamuzzic & Julien Marchal
timothykomadowski
Timk
#OneDegreeMore #perpetualmotion #perpetualmotionmachine #freeenergy #fuel #fuelprices #magnetmotor #magnet # генератор #bojacksonsmyhero
240 лайков, 45 комментариев. Видео TikTok от Timk (@timothykomadowski): «#OneDegreeMore #perpetualmotion #perpetualmotionmachine #freeenergy #fuel #fuelprices #magnetmotor #magnet #generator #bojacksonsmyhero». Мои грубые измерения показывают, что это менее 20 градусов, | 6 рычагов значительно уменьшают количество необходимой внешней силы. | Подписывайтесь, комментируйте,
Мол, начни спор! 🙃
С большой любовью💙. Все еще D.R.E. — Инструментальный.
11,6 тыс. просмотров|
Все еще Д.Р.Е. — Инструментальный — Dr. Dre
Магнитная энергия | AltEnergyMag
Вечный магнитный генератор невозможен? Возможно, но на эту теорию было выдано несколько патентов, и по мере того, как стоимость энергии продолжает расти, все больше ученых будут искать способы создать рабочий практичный вечный магнитный генератор.
Магнитная энергия
Лен Кальдероне
13. 06.12, 09:50
| Солнечный ветер
| Обсуждение технологий
Что, если бы вы вышли из дома и сели в машину будущего, похожую на пончик? Вы нажимаете кнопку, и автомобиль поднимается примерно на фут от земли. Небольшой двигатель двигает автомобиль вперед, когда он следует по электромагнитной полосе, встроенной в дорогу, при этом электромагнетизм отталкивает автомобиль от дороги.
Очевидно, что это всего лишь концепт-кар, представленный Volkswagen в Китае в рамках проекта People’s Car. Автомобиль VW Hover был представлен на автосалоне в Пекине в 2012 году.
Сегодня существуют электромагнитные двигатели и двигатели с постоянными магнитами, которые осуществимы и используются. Большое противоречие заключается в том, существует ли такая вещь, как магнитный двигатель (генератор) с вечным источником энергии.
Электромагнитная энергия — это энергия электромагнитного излучения, такого как радиоволны и волны видимого света, которые вызывают как электрические, так и магнитные поля. Компонент, который мы называем постоянным магнитом, представляет собой кусок магнитного материала, который после намагничивания или «зарядки» внешним магнитным полем сохраняет полезный большой магнитный момент после устранения намагничивающей силы. Таким образом, постоянный магнит сам становится источником магнитного поля, которое может взаимодействовать с другими намагничиваемыми материалами или с электрическими токами.
Простейшая форма магнитной энергии — фонарик Фарадея, который мы все видели. Вы встряхиваете фонарик вперед и назад, и действие создает энергию для питания лампочки. Принцип достаточно прост. Магнит проходит вперед и назад через катушку провода и создает электрический ток, который затем накапливается в конденсаторе. Когда фонарик включен, конденсатор подает накопленную энергию в лампочку, как в фонаре с батарейным питанием.
В основном эта система состоит из пяти частей. Магнит — это то, что генерирует энергию, когда она проходит через проволочную катушку. Чем сильнее магнит, тем больше энергии генерируется при каждом встряхивании. Размер проволочной катушки (количество витков) также определяет, сколько энергии вырабатывается при каждом проходе магнита. Конденсатор хранит энергию, которую вы генерируете, встряхивая фонарик. Чем выше качество и больше размер конденсатора, тем дольше световой поток. Тогда есть светодиодная лампа, которая имеет сниженное энергопотребление и срок службы. Наконец, есть кнопка включения/выключения.
Вопрос: «Можно ли создать вечный двигатель, используя аналогичный процесс?» Вечный двигатель в замкнутой системе нарушает первый закон термодинамики. Машины, производящие работу и энергию без затрат энергии, противоречат закону сохранения энергии. По законам термодинамики энергия не может просто создаваться или уничтожаться. Следовательно, настоящий вечный двигатель может никогда не стать жизнеспособным, но можно построить его близкую замену. В то время как энергия нужна для инициализации краткосрочного запуска вечного двигателя, что-то простое, например, рукоятка, может стать катализатором в устройстве, которое производит достаточно энергии, чтобы поддерживать себя и обеспечивать дополнительную мощность.
В этом типе двигателя используется конструкция с постоянными магнитами, в которой роторы удерживают постоянные магниты, расположенные вокруг вала. Эти магниты должны синхронизироваться с магнитами статора; а для создания хорошей мощности нужны редкоземельные элементы. Без разумного запаса материала постоянного магнита постоянные магниты не были бы очень постоянными. Проблема в том, что большая часть редкоземельных материалов, необходимых для изготовления надежных магнитов с длительным сроком службы, поступает из Китая.
Ниже приведен пример двигателя с постоянными магнитами и асинхронного двигателя с электромагнитным полем.
В асинхронном двигателе с электромагнитным полем вокруг статора создается вращающееся магнитное поле, которое вращается с синхронной (возникающей одновременно) скоростью. Это вращающееся магнитное поле проходит через воздушный зазор и перерезает неподвижные проводники ротора. Из-за относительной скорости между неподвижными проводниками ротора и вращающимся магнитным полем в проводниках ротора индуцируется электромагнитное поле. Когда проводники ротора замыкаются накоротко, через них начинает течь ток. И поскольку эти токонесущие проводники ротора помещены в магнитное поле, создаваемое статором, они испытывают механическую силу, которая перемещает ротор в том же направлении, что и вращающееся магнитное поле.
Двигатель с постоянными магнитами является разновидностью электродвигателя. В основном все типы двигателей работают, когда они имеют корпус статора и ротор. Во многих электродвигателях в качестве ротора используется электромагнит. В двигателе с постоянными магнитами ротор содержит постоянный магнит, а не электромагнит.
Двигатель с постоянными магнитами способен генерировать более высокий крутящий момент по сравнению с асинхронным двигателем. Кроме того, двигатель с постоянными магнитами можно использовать для производства электроэнергии, а не механического движения, особенно в ветроэнергетическом устройстве.
Магниты в двигателе с постоянными магнитами сделаны в основном из неодима и поэтому являются чрезвычайно мощными и долговечными постоянными магнитами. Для выработки электроэнергии ветер вращает турбину, которая затем включает магниты генератора и создает электрический ток. В результате при преобразовании кинетической формы энергии ветра в электрический ток фактически теряется гораздо меньше энергии.
XEMC Darwind производит высокопроизводительные ветряные турбины мощностью несколько мегаватт на основе технологии генератора с постоянными магнитами с прямым приводом.
Есть еще одно применение магнитов для создания эффективной энергии. Магнитогидродинамическая генерация энергии основана на законе электромагнитной индукции Фарадея. То есть, когда проводящая жидкость, такая как плазма, течет через магнитное поле, ионы будут двигаться в направлении, перпендикулярном как магнитному полю, так и направлению потока, и тогда возникнет электродвижущая сила. На сегодняшний день МГД является наиболее эффективной солнечной электрической технологией.
Слово Magneto Hydro Dynamic (MHD) происходит от Magneto, означающего магнитное поле, Hydro, означающего жидкость, и Dynamics, означающего движение.
Изображенный здесь МГД генерирует электричество непосредственно из тела очень горячего движущегося ионизированного газа без каких-либо механических движущихся частей. Солнечная энергия, сконцентрированная зеркалами и линзами, создает перегретые газы. Из-за более высокой температуры генерируемая солнечная МГД более эффективна, чем другие типы солнечных тепловых технологий, которые работают при гораздо более низкой температуре.
Магнитогидродинамика работает, используя сверхпроводящие магниты для извлечения электричества из перегретого движущегося ионизированного газа. В технологии МГД использование чрезвычайно больших сверхпроводящих постоянных магнитов повышает эффективность.
Первоначально генераторы с постоянными магнитами производят электричество, прикрепляя рукоятку или турбину, которая инициирует его движение. Ручная рукоятка будет использоваться бытовыми генераторами, в то время как турбина нужна генераторам, которые управляют гидроэлектростанциями. Магниты внутри генератора создают магнитное поле, которое активирует электричество в проводнике каждый раз, когда он проходит через него. Последовательное движение проводника создает устойчивый поток электричества.
Тем не менее, как с электромагнитными двигателями, так и с двигателями с постоянными магнитами, для запуска двигателя необходим внешний источник. Концепция вечного магнитного двигателя существует уже давно, но пока этот источник энергии не является жизнеспособным.
Идея магнитных вечных двигателей достаточно проста для понимания. Магнитные вечные двигатели приводятся в движение магнитами, которые заставляют вращаться пластины, и это движение приводит в движение генератор. Он может производить энергию или электроэнергию без необходимости в каком-либо внешнем источнике топлива. Электромагнитное поле, создаваемое расположением магнитов, является основой мощности, и как только генератор заработает, вы получите всю необходимую вам электроэнергию абсолютно бесплатно. Электрогенераторы, которые вы обычно находите в доме, требуют источника топлива, чтобы они могли производить электричество.
Принцип работы магнитного вечного двигателя заключается в том, что роторы приводятся в движение точно расположенными магнитами, а вращение роторов питает магнитный генератор так же, как ветрогенератор получает энергию от вращающегося ротора. Все эти моторы хоть и называются вечными, но таковыми не являются. Все изнашивается в какой-то момент времени, и магниты в конечном итоге разряжаются. По сути, вечный двигатель — это двигатель, который работает в течение длительного периода времени.
Энергия двигателя с вечным магнитом генерирует энергию из магнитных полей внутри магнитов. Эти поля можно использовать для инициирования силы, которая, в свою очередь, создает движение. Затем это движение может быть использовано для создания энергии.
Генератор с магнитным питанием — это другое название вечного магнитного генератора. Двигатели воспринимают силу, создаваемую полями внутри магнитов, и преобразуют эту силу в электрическую энергию.
Если вы возьмете достаточное количество магнитов и правильно расположите их, они будут отталкиваться друг от друга. Разместив эти магниты в форме круга, вы теоретически создадите колесо, которое будет вращаться, поскольку магнитные поля толкают колесо. Вращение колеса — это то, как двигатель вырабатывает энергию. Поскольку энергии в магнитах хватает на многие годы, колесо может вращаться и продолжать вращаться без необходимости когда-либо останавливаться, таким образом, движение вращающегося колеса создает энергию на многие годы. Это то, что превращает генератор с магнитным питанием в вечный генератор.
Двигатель с вечным магнитом Johnson патент номер 4151431
Невозможно? Возможно, но на эту теорию было выдано несколько патентов, и по мере того, как стоимость энергии продолжает расти, все больше ученых будут искать способы создать рабочий практичный вечный магнитный генератор.
Для получения дополнительной информации:
http://www.levitationfun.com/mfield.pdf
http://www.smma.org/mmpa_pmg-88.pdf
http://askmar.com/Magnets/Modern%20Permanent%20Magnet%20Applications.pdf
http://freeenergynews.com/Directory/Howard_Johnson_Motor/1979Paper/
О Лене
Лен начал работать в аудиовизуальной индустрии в 1975 году и написал статьи для нескольких изданий. Он также пишет редакционные статьи для местной газеты. Сейчас он на пенсии.
Эта статья содержит заявления личного мнения и комментарии, сделанные добросовестно в интересах общественности. Вы должны подтвердить все заявления с производителем, чтобы убедиться в правильности заявлений.
Содержание и мнения в этой статье принадлежат автору и не обязательно отражают точку зрения AltEnergyMag
13.06.12, 09:50
| Солнечный ветер
| Обсуждение технологий
Другие статьи о солнечной и ветровой энергии | Истории | Новости
Комментарии (0)
Этот пост не имеет комментариев. Будьте первым, кто оставит комментарий ниже.
Опубликовать комментарий
Вы должны войти в систему, прежде чем сможете оставлять комментарии. Войти сейчас.
Рекомендуемый продукт
Solis75 — Высокомощные линейные инверторы мощностью 100 кВт
Solis75kW — Мощные струнные инверторы серии 100kW широко используются в промышленных и коммерческих крышах. Новый трехфазный инвертор Solis — лучший выбор для промышленных и коммерческих фотоэлектрических систем. Трехфазный рядный инвертор мощностью 75–100 кВт имеет до 28 входных строк, поддерживает соединение типа «Y» на стороне постоянного тока. Максимальный входной ток строки 13 А, поддержка двустороннего доступа к модулям. Поддерживает функцию анти-ПИД для повышения эффективности системы.
Машина ошибок Джозефа Ньюмана
Эта статья появилась в октябре 1999 года в выпуске Phactum, бюллетеня Филадельфийской ассоциации критического мышления. Это авторское право © PhACT. Все права защищены.
— Том Нэпьер
Радиошоу Эрика Крига в Аризоне было реакцией на длительную кампанию Джозефа Ньюмана, направленную на то, чтобы провозгласить себя и его «энергетическую машину» спасителями мира. Вот некоторая предыстория для читателей, незнакомых с идеями Ньюмана. Я надеюсь, что в конечном счете я смогу рассмотреть их более подробно с технической точки зрения на веб-сайте PhACT.
С начала семидесятых годов Джо Ньюман продвигал электродвигатель, который, как он утверждает, вырабатывает больше энергии, чем питание от аккумуляторной батареи. Большую часть этого времени он боролся с патентным бюро. Они, вполне естественно, отказались выдать патент на то, что казалось вечным двигателем. Чтобы противостоять им, Ньюман засыпал научные журналы, конгрессменов и нескольких президентов США письмами с жалобами. Он даже баллотировался в президенты. Время от времени он устраивает демонстрации, чтобы побудить инвесторов финансировать более крупные и улучшенные версии его устройства.
Наш старый друг Деннис Ли научился бизнесу свободной энергии у Ньюмана. Оба утверждают, что были вдохновлены Богом, но только Ньюман утверждает, что его предсказал Нострадамус!
Большая часть того, что здесь появляется, взято из собственного выдающегося произведения Ньюмана «Энергетическая машина Джозефа Ньюмана». Эта книга размером 9 дюймов на 11 дюймов имеет толщину 1 3/4 дюйма, содержит около 590 страниц и переплетена из белой искусственной кожи с золотым тиснением названия. в терминах «гироскопических частиц». Есть описания его машины, жалобы на то, что патентное ведомство не тестирует ее должным образом, множество вырезок из прессы и перепечатки ни к чему не обязывающих писем, которые Ньюман получил от людей, к которым он приставал. простирается от статики через электромагнитную теорию до термодинамики и орбитальной механики.
Первые несколько страниц книги демонстрируют отношение, обычное для лжеученых. «Если я не могу понять общепринятую теорию, значит, она неверна». Ньюман считает, что в правиле, связывающем индукционный ток в проводнике с его движением в магнитном поле, есть изъян. Этот недостаток, по его словам, объясняет, почему обычные двигатели имеют КПД ниже 100%. *
Ньюман убедил себя, что магнитные поля состоят из частиц, которые ведут себя как крошечные гироскопы. Из этой концепции, используя ложную логику и без математических рассуждений, он заключает, что напряженность магнитного поля зависит не от тока, протекающего в катушке, а от массы меди. Затем он применяет к этой массе E=mc2, чтобы вычислить энергию поля, и заявляет, что эта энергия может быть высвобождена его машиной.
Одно из удивительных открытий Ньюмана состоит в том, что катушки с большим количеством витков провода генерируют заданное магнитное поле с меньшим входным током, чем катушки с меньшим количеством витков. Ну, даже Майкл Фарадей знал это. Далее он утверждает, что увеличение числа витков снижает потребляемую мощность. В этом расчете он игнорирует сопротивление провода. Кажется, Ньюман упустил из виду, что статическое магнитное поле не требует энергии для его поддержания. Только сопротивление медного провода рассеивает энергию. Например, сверхпроводящему магниту требуется входная мощность для создания поля, но не для его поддержания.
Ньюман намотал соленоиды с многотысячными витками провода. Они весили от 140 до 4200 фунтов. Требуются тысячи вольт, чтобы пропустить хоть какой-то заметный ток, поэтому он питает их от сотен 9-вольтовых батарей, соединенных последовательно. В его энергетической машине вращающийся магнит поворачивает коммутатор, который сложным образом включает и выключает ток в катушке. Возникающий в результате пульсирующий ток затрудняет измерение средней входной мощности. Однако Ньюман и его бывший помощник доктор Роджер Гастингс усугубили эту проблему, допустив элементарную ошибку при вычислении средней мощности. Это создает впечатление, что в катушке выделяется больше тепла, чем отдается батареей.
Другими словами, Ньюман не получает свою «сверхединичную» эффективность, сравнивая электрическую входную и механическую выходную мощность, как это сделал бы любой другой. Он просто ошибается в расчетах резистивных потерь, которые остальной мир будет рассматривать как потраченную впустую входную мощность, а затем прибавляет их к выходной мощности. Более того, он заявляет о мощности на высоких частотах, которую никто другой не может измерить, и он также зажигает люминесцентные лампы импульсами двигателя, чтобы показать его большую выходную мощность. Как и во многих схемах «сверхединицы», заявленный эффект возникает из-за ошибок измерения и неверных расчетов, а не из-за новых физических законов. Конечно, ни одно сверхединичное устройство еще не прошло окончательных испытаний, будучи способным бесконечно управлять собственным вводом.
Небольшое запутывание всегда помогает. Ньюман превозносит крошечный ток, потребляемый его двигателями, говоря людям, что он может управлять автомобилем (со скоростью пешехода) с транзисторной батареей для радиоприемника. Тот факт, что в машине было 1500 аккумуляторов, преуменьшается, как и несколько сотен ватт, которые они могут обеспечить даже при небольшом потреблении тока.
Еще до того, как я прочитал его книгу, я знал, основываясь на простых физических принципах, что машина Ньюмена не может производить больше энергии, чем в нее вложено. Однако я был ошеломлен, когда увидел фотографии двигателя, которые Ньюман представил в патентное бюро. Он состоит из ротора с постоянными магнитами и коллектора на валу. Рядом с ним стоит соленоид с воздушным сердечником. Теперь первым принципом создания эффективного электродвигателя является обеспечение прямого магнитного пути через якорь и катушки возбуждения. Обычно используется ферромагнитный сердечник, а все воздушные зазоры делаются настолько малыми, насколько это практически возможно. Ни один двигатель, состоящий из вращающегося магнита с катушкой рядом с ним, не может иметь КПД более нескольких процентов.
Даже Ньюман понял это. Его более поздние двигатели, о чем свидетельствует двигатель, построенный для него Нормом Биссом, состоят из обычного вращающегося якоря внутри стальной трубы с постоянными магнитами. От оригинальной конструкции остались только странный коммутатор и высоковольтные батареи. Увы, даже этот двигатель не может выдать больше мощности, чем он получает от аккумуляторов, но ему удается убедить наивных инвесторов раскошелиться.
* Обратите внимание на ошибочные рассуждения Ньюмана. Традиционная электрическая теория прекрасно объясняет наблюдаемое поведение всех двигателей, включая двигатель Ньюмена. Принятие неверной теории Ньюмана не может волшебным образом заставить двигатели работать по-другому. В любом случае нет необходимости модифицировать теорию до тех пор, пока не появятся экспериментальные основания полагать, что она ошибочна. Ньюман не представил никаких таких оснований.
электромагнетизм — Изготовление электродвигателя с использованием неодимовых магнитов?
Спросил
Изменено
1 месяц назад
Просмотрено
3к раз
$\begingroup$
(Хорошо, этот вопрос может показаться тривиальным для всех вас, умных физиков, но оставайтесь со мной и поделитесь своим мнением)
В последнее время я видел много видео в Интернете о том, как вы можете сделать генератор с помощью постоянного тока вентилятора и приклеив неодимовые магниты, обращенные в одном направлении, к лопастям вентилятора,
. Всякий раз, когда вы размещаете рядом с ним большой магнит, сила отталкивания магнитов заставит вентилятор постоянно двигаться и вращаться. Вентилятор в движении вращал двигатель и мог генерировать небольшое количество тока.
Многие физики в Интернете говорили, что это невозможно, а видео фальшивые. Мне стало любопытно, и я захотел попробовать, поэтому повторил эксперимент, но безуспешно . Я не очень разбираюсь в физике магнитов, но разве это «теоретически» не сработает? Я имею в виду, что вы не можете получить из него свободную энергию в соответствии с законом сохранения энергии и массы, но можно ли использовать магниты, чтобы заставить двигатель вращаться? Почему невозможно?
Если этот метод невозможен, возможно, есть другой способ, которым он мог бы работать?
- электромагнетизм
- магнитные поля
- законы сохранения
$\endgroup$
2
$\begingroup$
Реквизит для того, чтобы попробовать самому! Однако вы в основном ответили на свой вопрос — это невозможно, потому что вращение вентилятора с помощью статического магнита нарушит сохранение энергии.
Основная проблема заключается в том, что если вы заставите его вращаться, каждый магнит какое-то время будет притягиваться к магниту в вашей руке и какое-то время отталкиваться от него. В среднем это гаснет.
Если вы правильно рассчитали время, вы могли бы заставить его вращаться, перемещая магнит в руке. Требуемая энергия исходит от работы, которую вы выполняете рукой, так что здесь нет нарушения сохранения энергии.
$\endgroup$
$\begingroup$
сделать двигатель «свободной энергии» или «сверхединицы» с использованием только постоянных магнитов физически невозможно, и причины этого известны всем, кроме людей, которые снимают видео на YouTube, подобные тому, что вы видели. кроме того, невозможно также заставить идею работать каким-либо другим методом — в этом случае никакая хитрость не может преодолеть основную физику.
$\endgroup$
1
$\begingroup$
, поскольку вы уже сделали это и попробовали, вы можете попробовать следующее для лучшего понимания:
есть стекла/жидкости/материалы, которые могут визуализировать магнитные поля. наденьте один на свой рисунок и наблюдайте за формой магнитных полей. Основная проблема с такой конструкцией, если смотреть только на ее идею без использования каких-либо законов, говорящих о том, что определенные вещи невозможны, заключается в том, что магнитные поля имеют тенденцию быть довольно ровными / округленными. это означает, что перед тем, как магнит на вентиляторе пройдет мимо магнита, который вы держите, ему нужно вытолкнуть столько энергии, сколько он получит позже, в общем. это тот же принцип, который делает магнитные шестерни такими эффективными, поскольку они не имеют физического контакта, а работают исключительно за счет этих сил, поэтому энергия, которую они теряют (исключая сопротивление среды, квантовые эффекты, трение в подшипниках и другие подобные вещи), они будут также получить обратно. такая шестерня может зафиксироваться в предпочтительном состоянии из-за неправильного изготовления, но вся энергия возвращается, как только она начинает двигаться (энергия теряется из-за магнитного толчка).